Линейная скорость формула через число оборотов: Линейная скорость через угловую, теория и онлайн калькуляторы

Содержание

1.9.2 Угловая и линейная скорости вращения

Вращательное движение вокруг неподвижной оси — еще один частный случай движения твердого тела.


Вращательным движением твердого тела вокруг неподвижной оси называется такое его движение, при котором все точки тела описывают окружности, центры которых находятся на одной прямой, называемой осью вращения, при этом плоскости, которым принадлежат эти окружности, перпендикулярны оси вращения (рис.2.4).
В технике такой вид движения встречается очень часто: например, вращение валов двигателей и генераторов, турбин и пропеллеров самолетов.
Угловая скорость. Каждая точка вращающегося вокруг оси тела, проходящей через точку О, движется по окружности, и различные точки проходят за время разные пути. Так, , поэтому модуль скорости точки А больше, чем у точки В (рис.2.5). Но радиусы окружностей поворачиваются за время на один и тот же угол . Угол — угол между осью ОХ и радиус-вектором , определяющим положение точки А (см. рис.2.5).
Пусть тело вращается равномерно, т. е. за любые равные промежутки времени поворачивается на одинаковые углы. Быстрота вращения тела зависит от угла поворота радиус-вектора, определяющего положение одной из точек твердого тела за данный промежуток времени; она характеризуется угловой скоростью. Например, если одно тело за каждую секунду поворачивается на угол , а другое — на угол , то мы говорим, что первое тело вращается быстрее второго в 2 раза.
Угловой скоростью тела при равномерном вращении называется величина, равная отношению угла поворота тела к промежутку времени , за который этот поворот произошел.
Будем обозначать угловую скорость греческой буквой ω (омега). Тогда по определению
Угловая скорость выражается в радианах в секунду (рад/с).
Например, угловая скорость вращения Земли вокруг оси равна 0,0000727 рад/с, а точильного диска — около 140 рад/с1.
Угловую скорость можно выразить через частоту вращения, т. е. число полных оборотов за 1с. Если тело совершает (греческая буква «ню») оборотов за 1с, то время одного оборота равно секунд. Это время называют периодом вращения и обозначают буквой T. Таким образом, связь между частотой и периодом вращения можно представить в виде:
Полному обороту тела соответствует угол . Поэтому согласно формуле (2.1)
Если при равномерном вращении угловая скорость известна и в начальный момент времени угол поворота , то угол поворота тела за время t согласно уравнению (2.1) равен:
Если , то , или .
Угловая скорость принимает положительные значения, если угол между радиус-вектором, определяющим положение одной из точек твердого тела, и осью ОХ увеличивается, и отрицательные, когда он уменьшается.
Тем самым мы можем описать положение точек вращающегося тела в любой момент времени.
Связь между линейной и угловой скоростями. Скорость точки, движущейся по окружности, часто называют линейной скоростью, чтобы подчеркнуть ее отличие от угловой скорости.
Мы уже отмечали, что при вращении твердого тела разные его точки имеют неодинаковые линейные скорости, но угловая скорость для всех точек одинакова.
Между линейной скоростью любой точки вращающегося тела и его угловой скоростью существует связь. Установим ее. Точка, лежащая на окружности радиусом R, за один оборот пройдет путь . Поскольку время одного оборота тела есть период T, то модуль линейной скорости точки можно найти так:
Так как , то
Из этой формулы видно, что, чем дальше расположена точка тела от оси вращения, тем больше ее линейная скорость. Для точек земного экватора , а для точек на широте Санкт-Петербурга . На полюсах Земли .
Модуль ускорения точки тела, движущейся равномерно по окружности, можно выразить через угловую скорость тела и радиус окружности:
Следовательно,
Чем дальше расположена точка твердого тела от оси вращения, тем большее по модулю ускорение она имеет.
Итак, мы научились полностью описывать движение абсолютно твердого тела, вращающегося равномерно вокруг неподвижной оси, так как, пользуясь формулами , можем находить положение, модули скорости и ускорения любой точки тела в произвольный момент времени. Знаем мы и направления и , a также форму траекторий точек.

Вращательное движение

Вращательное движение является периодическим движением.

Период обозначается буквой T.

Чтобы найти период обращения, надо время вращения разделить на число оборотов:

Физическая величина, равная отношению числа полных оборотов тела ко времени, в течение которого эти обороты совершены, называется частотой вращения.

Частота вращения обозначается буквой n.

Чтобы найти частоту вращения, надо число оборотов разделить на время, в течение которого эти обороты совершены:

Частота вращения и период обращения связаны друг с другом как взаимообратные величины: Период измеряется в секундах: [T] = 1 с.

Единица частоты – секунда в минус первой степени: [n] = 1 с–1.

Эта единица имеет собственное название – 1 герц (1 Гц).

Проведем аналогию между вращательным и поступательным движениями.

Поступательно движущееся тело изменяет свое положение в пространстве относительно других тел.

Тела, совершающие вращательное движение поворачиваются на некоторый угол.

Если за любые равные промежутки времени поступательно движущееся тело совершает равные перемещения, движение называется равномерным.

Если за любые равные промежутки времени вращающееся тело поворачивается на один и тот же угол, то такое вращение называется равномерным. Характеристикой равномерного поступательного движения служит скорость Соответствующей характеристикой вращательного движения служит угловая скорость:

Угловая скорость – это физическая величина, равная отношению угла поворота тела ко времени, в течение которого этот поворот совершен.

Угловая скорость показывает, на какой угол поворачивается тело за единицу времени.

Чтобы получить единицу угловой скорости, нужно в ее определяющую формулу подставить единицу – 1 радиан, и времени – 1 с. Получаем: [ω] = 1

Аналогично можно ввести характеристику неравномерного вращения. Если видом неравномерного поступательного движения является равнопеременное движение, то для вращательного движения можно ввести понятие равнопеременного вращения.

Характеристикой равнопеременного поступательного движения является ускорение:

Соответственно, для вращательного движения можно ввести величину, определяемую отношением изменения угловой скорости ко времени, в течение которого это изменение происходит – угловое ускорение: Угловое ускорение показывает, на сколько изменилась угловая скорость за единицу времени.

Чтобы получить единицу углового ускорения, нужно в его определяющую формулу подставить единицы угловой скорости 1 рад/с и времени – 1 с. Получаем:

Продолжая аналогию дальше, запишем уравнение для перемещения при прямолинейном равноускоренном движении

Так как при вращении перемещению тела соответствует угол вращения, линейной скорости – угловая скорость, линейному ускорению – угловое ускорение, то аналогичное уравнение для вращательного движения будет иметь вид:

Другому уравнению для поступательного движения будет соответствовать уравнение для вращательного движения:

Метод, который использовался в данном случае, называется методом аналогий.

Точки тела, совершающего вращательное движение, поворачиваются относительно оси вращения на некоторые углы и движутся по дугам окружностей, проходя определенные пути. Таким образом, характеристиками вращательного движения являются и угловая, и линейная скорости.

Линейная скорость точки направлена по касательной к окружности, по которой она движется.

Об этом свидетельствует слетающая с колес автомобиля грязь или искры, летящие от металлического предмета, прижатого к наждачному кругу.

Чем дальше от оси вращения находится точка, тем больше ее линейная скорость. Угловая же скорость точек, лежащих на одном радиусе, одинакова. Следовательно, линейная скорость точки прямо пропорциональна радиусу окружности, по которой она вращается.

За время, равное периоду, точка проходит путь, равный длине окружности. Её линейная скорость при этом равна Отношение же угла поворота ко времени поворота на этот угол равно угловой скорости

Таким образом, линейная скорость вращающейся точки связана с ее угловой скоростью соотношением:

При равномерном вращении скорость меняется по направлению, но не изменяется по величине.

Поскольку всякое изменение скорости характеризуется понятием ускорения, то для равномерного вращения следует ввести еще одно ускорение, изменяющееся не по величине, а по направлению. Это ускорение называют центростремительным.

Пусть вращающееся тело в начальный момент времени находится в точке A и скорость его направлена по касательной. В следующий момент времени тело находится в точке B. При этом скорость его изменилось только по направлению и направлена по касательной к окружности.

Найдем вектор разности скоростей, воспользовавшись правилом действия с векторами. Из чертежа видно, что вектор разности направлен в сторону близкую к центру окружности. Чем меньше угол поворота, тем ближе направлен вектор скорости к направлению на центр вращения.

При малом времени движения изменение положения тела незначительно. Поэтому можно считать, что вектор скорости характеризующий изменение скорости по направлению, направлен на центр. Отсюда и происходит название центростремительного ускорения.

Угловое же ускорение, характеризующее изменение скорости по величине, называют еще касательным или тангенциальным ускорением (при неравномерном вращении).

Получим выражение для центростремительного ускорения. Будем считать, что угол поворота очень мал. Соединим точки A и B. Угол MAN = φ по построению.

Мы имеем два равнобедренных треугольника. Треугольник OAB, ребра которого R и AB, и треугольник MAN, ребра которого и

Так как треугольники подобны (по двум сторонам и углу между ними), то можно записать:

Дуга окружности и хорда практически равны из-за малости угла поворота. Поэтому дуга Следовательно, Получим

Разделив правую и левую части последнего уравнения на t, получим:

Отсюда Таким образом,

Полученная формула является формулой для расчета центростремительного ускорения.

Центростремительное ускорение, при движении тела по окружности, равно отношению квадрата скорости к радиусу окружности, по которой движется тело:

Урок 5. поступательное движение. вращательное движение твердого тела — Физика — 10 класс

Физика, 10 класс

Урок 05. Поступательное движение. Вращательное движение твёрдого тела

Перечень вопросов, рассматриваемых на уроке:

  1. Поступательное и вращательное движение абсолютно твердого тела.
  2. Характеристики вращательного движения абсолютно твердого тела.

Глоссарий по теме

1. Абсолютно твердое тело – это тело, расстояние между любыми двумя точками которого остается постоянным при его движении.

2. Поступательным называется такое движение абсолютно твердого тела, при котором любой отрезок, соединяющий любые две точки тела, остается параллельным самому себе. Одинаковыми остаются при поступательном движении перемещение, траектория, путь, скорость, ускорение.

3. Вращением твердого тела вокруг неподвижной оси называется такое движение, при котором все точки тела описывают окружности, центры которых находятся на одной прямой перпендикулярной плоскостям этих окружностей. Сама эта прямая есть ось вращения.

4. Угол поворота – угол, на который поворачивается радиус-вектор, соединяющий центр окружности с точкой вращающегося тела.

5. Угловая скорость — отношение угла поворота φ к промежутку времени, в течение которого совершен этот поворот при равномерном движении.

6. Линейная скорость – отношение длины дуги окружности пройденной точкой тела к промежутку времени, в течение которого этот поворот совершен.

7. Период — промежуток времени, за который тело делает один полный оборот.

8. Частота обращения тела – число оборотов за единицу времени

Основная и дополнительная литература по теме урока:

Мякишев Г.Я., Буховцев Б.Б., Сотский Н.Н.. Физика.10 класс. Учебник для общеобразовательных организаций М.: Просвещение, 2016. – С. 57-61

Рымкевич А.П. Сборник задач по физике. 10-11 класс.-М.:Дрофа,2009.-С.20-22

Открытые электронные ресурсы:

http://kvant.mccme.ru/1986/11/kinematika_vrashchatelnogo_dvi.htm

Теоретический материал для самостоятельного изучения

1. Вы знаете, что в физике для упрощения исследования реальных ситуаций часто используются модели. Одной из механических моделей, используемых при описании движения и взаимодействия тел, является абсолютно твёрдое тело- тело, расстояние между любыми двумя точками которого остаётся постоянным при его движении.

2. Поступательным называется такое движение абсолютно твёрдого тела, при котором любой отрезок, соединяющий любые две точки тела, остаётся параллельным самому себе. Примером поступательного движения может служить свободное падение тел, движение лифта, поезда на прямолинейном участке дороги. При поступательном движении все точки тела описывают одинаковые траектории, совершают одинаковые перемещения, проходят одинаковые пути, в каждый момент времени имеют равные скорости и ускорения.

Для описания поступательного движения абсолютно твёрдого тела достаточно написать уравнение движения одной из его точек.

3. Вращательным движением абсолютно твёрдого тела вокруг неподвижной оси называется такое его движение, при котором все точки тела описывают окружности, центры которых находятся на одной прямой, называемой осью вращения. При этом плоскости, которым принадлежат эти окружности, перпендикулярны оси вращения.

Вращательное движение позволяет осуществить непрерывный процесс работы с использованием больших скоростей. Вращающиеся механизмы более компактны и более экономичны, так как потери энергии на преодоление сил трения качения меньше, чем на преодоление сил трения скольжения. Поэтому в современной технике вращательное движение рабочих частей машин всё более вытесняет возвратно-поступательное. Например, вместо ножовочной пилы в технике используют вращающуюся дисковую пилу, поршневые насосы в большинстве случаев вытесняются центробежными.

4. Угловой скоростью тела при равномерном вращении называется величина, равная отношению угла поворота тела ∆φ к промежутку времени ∆t, за которое этот поворот произошёл.

Будем обозначать угловую скорость греческой буквой ω (омега). Тогда по определению запишем формулу угловой скорости;

При равномерном вращательном движении угловая скорость у всех точек вращающегося тела одинаковая. Поэтому угловая скорость, так же как и угол поворота, является характеристикой движения всего вращающегося тела, а не только отдельных его частей.

Примером вращательного движения, близкого к равномерному, может служить вращение Земли вокруг своей оси.

Угловая скорость в СИ выражается в радианах в секунду (рад/с).

Один радиан – это центральный угол, опирающийся на дугу, длина которой равна радиусу окружности.

Угловая скорость положительна, если угол между радиусом вектором, определяющим положение одной из точек твердого тела, и осью ОХ увеличивается, и отрицательным, когда он уменьшается

5.Число полных оборотов за единицу времени называют частотой обращения.

Частоту обозначают греческой буквой «ню». Единица измерения частоты является секунда в минус первой степени

Время, за которое тело совершает один полный оборот, называют периодом обращения и обозначают буквой Т.

7. Связь между линейной и угловой скоростями:

8. Связь между ускорением и угловой скоростью:

Итак, мы рассмотрели два простейших движения абсолютно твердого тела – поступательное и вращательное. В жизни мы чаще встречаем сложное движение абсолютно твердого тела, однако, в этом случае любое сложное движение можно представить как сумму двух независимых движений: поступательного и вращательного.

Примеры и разбор типового тренировочного задания

  1. Ротор мощной паровой турбины делает 100 оборотов за 2 с. Определите угловую скорость.

Дано:

N=100 об.

t = 2 c

Найти: ω.

Решение:

2. Два шкива, соединенные друг с другом ремнем, вращаются вокруг неподвижных осей (см.рис). Больший шкив радиусом 20см делает 50 оборотов за 10 секунд, а частота вращения меньшего шкива 2400 оборотов в минуту. Чему равен радиус меньшего шкива? Шкивы вращаются без проскальзывания.

Дано:

Найти —

Решение:

Из условия задачи ученик видит что, шкивы соединены ремнем, следовательно, линейные скорости их равны:

но частота вращения разная.

Сокращает на 2π обе части.

Отсюда имеем:

и так, как в условии известно , то можем записать:

Отсюда находим радиус второго шкива:

Вторая неизвестная величина

Запишем формулу периода обращения для большего шкива:

так как по условию задачи нам известно число оборотов за 10 секунд.

Подставим в формулу (1) и получим конечную формулу:

7.2: Классическая механика

Область классической механики включает изучение тел в движении, особенно физические законы, касающиеся тел, находящихся под воздействием сил. Большинство механических аспектов проектирования роботов тесно связано с концепциями из этой области. В данном блоке описываются несколько ключевых применяемых концепций классической механики.

СКОРОСТЬ — это мера того, насколько быстро перемещается объект. Обозначает изменение положения во времени (проще говоря, какое расстояние способен преодолеть объект за заданный период времени). Данная мера представлена в единицах расстояния, взятых в единицу времени, например, в количестве миль в час или футов в секунду.

ЧАСТОТА ВРАЩЕНИЯ – Скорость может также выражаться во вращении, то есть насколько быстро объект движется по кругу. Измеряется в единицах углового перемещения во времени (то есть в градусах в секунду), или в циклах вращения в единицу времени (например, в оборотах в минуту). Когда измерения представлены в оборотах в минуту (RPM), речь идет о частоте вращения. Есть речь идет об об/мин автомобильного двигателя, это означает, что измеряется скорость вращения двигателя.

УСКОРЕНИЕ – Изменение скорости во времени представляет собой ускорение. Чем больше ускорение, тем быстрее изменяется скорость. Если автомобиль развивает скорость от 0 до 60 миль в час за две секунды, в этом случае ускорение больше, чем когда он развивает скорость от 0 до 40 миль в час за тот же период времени. Ускорение — это мера изменения скорости. Отсутствие изменения означает отсутствие ускорения. Если объект движется с постоянной скоростью — ускорение отсутствует.

СИЛА — Ускорение является следствием воздействия сил, которые провоцируют изменение в движении, направлении или форме. Если вы нажимаете на объект, это означает, что вы прикладываете к нему силу. Робот ускоряется под воздействием силы, которую его колеса прикладывают к полу. Сила измеряется в фунтах или ньютонах.

Например, масса объекта воздействует на объект как сила вследствие гравитации (ускорение объекта в направлении центра Земли).

КРУТЯЩИЙ МОМЕНТ – Сила, направленная по кругу (вращение объекта), называется крутящим моментом. Крутящий момент — это вращающая сила. Если к объекту приложен крутящий момент, на границе первого возникает линейная сила. В примере с колесом, катящемся по земле, крутящий момент, приложенный к оси колеса, создает линейную силу на границе покрышки в точке ее контакта с поверхностью земли. Так и определяется крутящий момент — как линейная сила на границе круга. Крутящий момент определяется величиной силы, умноженной на расстояние от центра вращения (Сила х Расстояние = Крутящий момент). Крутящий момент измеряется в единицах силы, умноженной на расстояние, например, фунто-дюймах или ньютон-метрах.

В примере с колесом, катящемся по земле, если известен крутящий момент, приложенный к оси с закрепленным на ней колесом, мы можем рассчитать количество силы, прикладываемой колесом к поверхности. В этом случае, радиус колеса является расстоянием силы от центра вращения.

Сила = Крутящий момент/Радиус колеса

В примере с рукой робота, удерживающей объект, мы можем рассчитать крутящий момент, требуемый для поднятия объекта. Если объект обладает массой, равной 1 ньютону, а рука имеет длину 0,25 метра (объект располагается на расстоянии 0,25 метра от центра вращения), тогда

Крутящий момент = Сила х Расстояние = 1 ньютон х 0,25 метра = 0,25 ньютон-метров.

Это означает, что для удержания объекта в неподвижном положении, необходимо применить крутящий момент, равный 0,25 ньютон-метров. Чтобы переместить объект вверх, роботу необходимо приложить к нему крутящий момент, значение которого будет превышать 0,25 ньютон-метров, так как необходимо преодолеть силу гравитации. Чем больше крутящий момент робота, тем больше силы он прикладывает к объекту, тем больше ускорение объекта, и тем быстрее рука поднимет объект.

Пример 7.2

Пример 7.3

Для данных примеров, мы можем рассчитать крутящий момент, необходимый для подъем этих объектов.

Пример 7.2 — Крутящий момент = Сила х Расстояние = 1 ньютон х 0,125 метра = 0,125 ньютон-метров.

Для данного примера, длина рука равна половине длины руки из Примера 1, поэтому значение требуемого крутящего момента также в два раза меньше. Значение длины руки пропорционально значению требуемого крутящего момента. При равных исходных характеристиках объекта, чем короче рука, тем меньший крутящий момент необходим для подъема.

Пример 7.3 — Крутящий момент = Сила * Расстояние = 1 ньютон х 0,5 метра = 0,5 ньютон-метров.

Для данного примера, длина рука равна удвоенной длине руки из Примера 1, поэтому значение требуемого крутящего момента также в два раза больше.

Еще одна точка зрения относительно ограниченного крутящего момента в соединении руки робота заключается в следующем: более короткая рука сможет поднять объект большей массы, чем более длинная рука; однако, для первой доступная высота подъема объекта будет меньше, чем для второй.

Пример 7.4

Пример 7.5

Эти примеры иллюстрируют руку робота, поднимающую объекты разной массы. Какова взаимосвязь с требуемым количеством крутящего момента?

Пример 4 — Крутящий момент = Сила х Расстояние = ½ ньютона х 0,25 метра = 0,125 ньютон-метров.

Пример 5 — Крутящий момент = Сила х Расстояние = 2 ньютона х 0,25 метра = 0,5 ньютон-метров.

Эти примеры иллюстрируют уменьшение значения требуемого крутящего момента по мере снижения массы объекта. Масса пропорциональна крутящему моменту, необходимому для ее подъема. Чем тяжелее объект, тем больше крутящий момент, требуемый для его подъема.

Проектировщики роботов должны обратить внимание на ключевые взаимосвязи между значениями крутящего момента, длины руки и массы объекта.

РАБОТА – Мера силы, приложенной на расстоянии, называется работой. Например, для удерживания объекта необходимо 10 фунтов силы. Далее, чтобы поднять этот объект на высоту 10 дюймов, требуется определенное количество работы. Количество работы, требуемое для подъема объекта на высоту 20 дюймов, удваивается. Работа также понимается как изменение энергии.

МОЩНОСТЬ — Большинство людей полагает, что мощность является термином из области электрики, но мощность также относится и к механике.

Мощность — это количество работы в единицу времени. Насколько быстро кто-то может выполнить работу?

В робототехнике принято понимать мощность как ограничение, так как соревновательные робототехнические системы имеют ограничения в части выходной мощности. Если роботу требуется поднять массу в 2 ньютона (прилагая 2 ньютона силы), скорость подъема будет ограничиваться количеством выходной мощности робота. Если робот способен произвести достаточное количество мощности, он сможет быстро поднять объект. Если он способен произвести лишь малое количество энергии, подъем объекта будет производиться медленно (либо не будет производиться вообще!).

Мощность определяется как Сила, умноженная на Скорость (насколько быстро выполняется толчок при постоянной скорости), и обычно выражается в Ваттах.

Мощность [Ватты] = Сила [Ньютоны] х Скорость [Метры в секунду]

1 Ватт = 1 (Ньютон х Метр) / Секунда

Как это применяется в соревновательной робототехнике? К проектам роботов применяются определенные ограничения. Проектировщики соревновательных роботов, использующие систему проектирования VEX Robotics Design, также должны учитывать физические ограничения, связанные с применением электромоторов. Электромотор обладает ограниченной мощностью, поэтому он может производить только определенное количество работы с заданной скоростью.

Примечание: все перспективные концепции имеют базовое описание. Более глубоко обсуждать эти физические свойства учащиеся будут в процессе обучения в ВУЗах, если выберут область STEM в качестве направления обучения.

 

Равномерное движение тела по окружности – FIZI4KA

1. Движением тела по окружности называют движение, траекторией которого является окружность. По окружности движутся, например, конец стрелки часов, точки лопасти вращающейся турбины, вращающегося вала двигателя и др.

При движении по окружности направление скорости непрерывно изменяется. При этом модуль скорости тела может изменяться, а может оставаться неизменным. Движение, при котором изменяется только направление скорости, а её модуль сохраняется постоянным, называется равномерным движением тела по окружности. Под телом в данном случае имеют в виду материальную точку.

2. Движение тела по окружности характеризуется определёнными величинами. К ним относятся, прежде всего, период и частота обращения. Период обращения тела по окружности ​\( T \)​ — время, в течение которого тело совершает один полный оборот. Единица периода — ​\( [\,T\,] \)​ = 1 с.

Частота обращения ​\( (n) \)​ — число полных оборотов тела за одну секунду: ​\( n=N/t \)​. Единица частоты обращения — \( [\,n\,] \) = 1 с-1 = 1 Гц (герц). Один герц — это такая частота, при которой тело совершает один оборот за одну секунду.

Связь между частотой и периодом обращения выражается формулой: ​\( n=1/T \)​.

Пусть некоторое тело, движущееся по окружности, за время ​\( t \)​ переместилось из точки А в точку В. Радиус, соединяющий центр окружности с точкой А, называют радиусом-вектором. При перемещении тела из точки А в точку В радиус-вектор повернётся на угол ​\( \varphi \)​.

Быстроту обращения тела характеризуют угловая и линейная скорости.

Угловая скорость ​\( \omega \)​ — физическая величина, равная отношению угла поворота \( \varphi \) радиуса-вектора к промежутку времени, за которое этот поворот произошел: ​\( \omega=\varphi/t \)​. Единица угловой скорости — радиан в секунду, т.е. ​\( [\,\omega\,] \)​ = 1 рад/с. За время, равное периоду обращения, угол поворота радиуса-вектора равен ​\( 2\pi \)​. Поэтому ​\( \omega=2\pi/T \)​.

Линейная скорость тела ​\( v \)​ — скорость, с которой тело движется вдоль траектории. Линейная скорость при равномерном движении по окружности постоянна по модулю, меняется по направлению и направлена по касательной к траектории.

Линейная скорость равна отношению пути, пройденному телом вдоль траектории, ко времени, за которое этот путь пройден: ​\( \vec{v}=l/t \)​. За один оборот точка проходит путь, равный длине окружности. Поэтому ​\( \vec{v}=2\pi\!R/T \)​. Связь между линейной и угловой скоростью выражается формулой: ​\( v=\omega R \)​.

Из этого равенства следует, что чем дальше от центра окружности расположена точка вращающегося тела, тем больше её линейная скорость.

4. Ускорение тела равно отношению изменения его скорости ко времени, за которое оно произошло. При движении тела по окружности изменяется направление скорости, следовательно, разность скоростей не равна нулю, т.е. тело движется с ускорением. Оно определяется по формуле: ​\( \vec{a}=\frac{\Delta\vec{v}}{t} \)​ и направлено так же, как вектор изменения скорости. Это ускорение называется центростремительным ускорением.

Центростремительное ускорение при равномерном движении тела по окружности — физическая величина, равная отношению квадрата линейной скорости к радиусу окружности: ​\( a=\frac{v^2}{R} \)​.2R \)​.

При движении тела по окружности его центростремительное ускорение постоянно по модулю и направлено к центру окружности.

ПРИМЕРЫ ЗАДАНИЙ

Часть 1

1. При равномерном движении тела по окружности

1) изменяется только модуль его скорости
2) изменяется только направление его скорости
3) изменяются и модуль, и направление его скорости
4) не изменяется ни модуль, ни направление его скорости

2. Линейная скорость точки 1, находящейся на расстоянии ​\( R_1 \)​ от центра вращающегося колеса, равна ​\( v_1 \)​. Чему равна скорость ​\( v_2 \)​ точки 2, находящейся от центра на расстоянии ​\( R_2=4R_1 \)​?

1) ​\( v_2=v_1 \)​
2) ​\( v_2=2v_1 \)​
3) ​\( v_2=0,25v_1 \)​
4) ​\( v_2=4v_1 \)​

3. Период обращения точки по окружности можно вычислить по формуле:

1) ​\( T=2\pi\!Rv \)​
2) \( T=2\pi\!R/v \)​
3) \( T=2\pi v \)​
4) \( T=2\pi/v \)​

4.2 \)​
3) \( \omega=vR \)
4) \( \omega=v/R \)​

5. Угловая скорость вращения колеса велосипеда увеличилась в 2 раза. Как изменилась линейная скорость точек обода колеса?

1) увеличилась в 2 раза
2) уменьшилась в 2 раза
3) увеличилась в 4 раза
4) не изменилась

6. Линейная скорость точек лопасти винта вертолёта уменьшилась в 4 раза. Как изменилось их центростремительное ускорение?

1) не изменилось
2) уменьшилось в 16 раз
3) уменьшилось в 4 раза
4) уменьшилось в 2 раза

7. Радиус движения тела по окружности увеличили в 3 раза, не меняя его линейную скорость. Как изменилось центростремительное ускорение тела?

1) увеличилось в 9 раз
2) уменьшилось в 9 раз
3) уменьшилось в 3 раза
4) увеличилось в 3 раза

8. Чему равен период обращения коленчатого вала двигателя, если за 3 мин он совершил 600 000 оборотов?

1) 200 000 с
2) 3300 с
3) 3·10-4 с
4) 5·10-6 с

9.2/R \)​
3) ​\( v/R \)​
4) ​\( \omega R \)​
5) ​\( 1/n \)​

12. Период обращения колеса увеличился. Как изменились угловая и линейная скорости точки обода колеса и её центростремительное ускорение. Установите соответствие между физическими величинами в левом столбце и характером их изменения в правом столбце.
В таблице под номером физической величины левого столбца запишите соответствующий номер выбранного вами элемента правого столбца.

ФИЗИЧЕСКАЯ ВЕЛИЧИНА
A) угловая скорость
Б) линейная скорость
B) центростремительное ускорение

ХАРАКТЕР ИЗМЕНЕНИЯ ВЕЛИЧИНЫ
1) увеличилась
2) уменьшилась
3) не изменилась

Часть 2

13. Какой путь пройдёт точка обода колеса за 10 с, если частота обращения колеса составляет 8 Гц, а радиус колеса 5 м?

Ответы

Равномерное движение тела по окружности

3 (60.87%) 23 votes

Конвертер угловой скорости и частоты вращения • Механика • Компактный калькулятор • Онлайн-конвертеры единиц измерения

Конвертер длины и расстоянияКонвертер массыКонвертер мер объема сыпучих продуктов и продуктов питанияКонвертер площадиКонвертер объема и единиц измерения в кулинарных рецептахКонвертер температурыКонвертер давления, механического напряжения, модуля ЮнгаКонвертер энергии и работыКонвертер мощностиКонвертер силыКонвертер времениКонвертер линейной скоростиПлоский уголКонвертер тепловой эффективности и топливной экономичностиКонвертер чисел в различных системах счисления.Конвертер единиц измерения количества информацииКурсы валютРазмеры женской одежды и обувиРазмеры мужской одежды и обувиКонвертер угловой скорости и частоты вращенияКонвертер ускоренияКонвертер углового ускоренияКонвертер плотностиКонвертер удельного объемаКонвертер момента инерцииКонвертер момента силыКонвертер вращающего моментаКонвертер удельной теплоты сгорания (по массе)Конвертер плотности энергии и удельной теплоты сгорания топлива (по объему)Конвертер разности температурКонвертер коэффициента теплового расширенияКонвертер термического сопротивленияКонвертер удельной теплопроводностиКонвертер удельной теплоёмкостиКонвертер энергетической экспозиции и мощности теплового излученияКонвертер плотности теплового потокаКонвертер коэффициента теплоотдачиКонвертер объёмного расходаКонвертер массового расходаКонвертер молярного расходаКонвертер плотности потока массыКонвертер молярной концентрацииКонвертер массовой концентрации в раствореКонвертер динамической (абсолютной) вязкостиКонвертер кинематической вязкостиКонвертер поверхностного натяженияКонвертер паропроницаемостиКонвертер плотности потока водяного параКонвертер уровня звукаКонвертер чувствительности микрофоновКонвертер уровня звукового давления (SPL)Конвертер уровня звукового давления с возможностью выбора опорного давленияКонвертер яркостиКонвертер силы светаКонвертер освещённостиКонвертер разрешения в компьютерной графикеКонвертер частоты и длины волныОптическая сила в диоптриях и фокусное расстояниеОптическая сила в диоптриях и увеличение линзы (×)Конвертер электрического зарядаКонвертер линейной плотности зарядаКонвертер поверхностной плотности зарядаКонвертер объемной плотности зарядаКонвертер электрического токаКонвертер линейной плотности токаКонвертер поверхностной плотности токаКонвертер напряжённости электрического поляКонвертер электростатического потенциала и напряженияКонвертер электрического сопротивленияКонвертер удельного электрического сопротивленияКонвертер электрической проводимостиКонвертер удельной электрической проводимостиЭлектрическая емкостьКонвертер индуктивностиКонвертер реактивной мощностиКонвертер Американского калибра проводовУровни в dBm (дБм или дБмВт), dBV (дБВ), ваттах и др. единицахКонвертер магнитодвижущей силыКонвертер напряженности магнитного поляКонвертер магнитного потокаКонвертер магнитной индукцииРадиация. Конвертер мощности поглощенной дозы ионизирующего излученияРадиоактивность. Конвертер радиоактивного распадаРадиация. Конвертер экспозиционной дозыРадиация. Конвертер поглощённой дозыКонвертер десятичных приставокПередача данныхКонвертер единиц типографики и обработки изображенийКонвертер единиц измерения объема лесоматериаловВычисление молярной массыПериодическая система химических элементов Д. И. Менделеева

Потолочный вентилятор, вращающийся со скоростью 250 оборотов в минуту

Общие сведения

Угловая скорость — это векторная величина, определяющая скорость вращения тела относительно оси вращения. Этот вектор направлен перпендикулярно плоскости вращения и определяется с помощью правила буравчика. Угловую скорость измеряют как отношение между углом, на который переместилось тело, то есть угловым смещением, и временем, на это потраченным. В системе СИ угловое ускорение измеряют в радианах в секунду.

Угловая скорость в спорте

Угловая скорость часто используется в спорте. Например, спортсмены уменьшают или увеличивают угловую скорость движения клюшки для гольфа, биты или ракетки, чтобы улучшить результаты. Угловая скорость связана с линейной скоростью так, что из всех точек на отрезке, вращающемся вокруг точки на этом отрезке, то есть вокруг центра вращения, самая отдаленная точка от этого центра движется с самой высокой линейной скоростью. Так, например, если клюшка для гольфа вращается, то конец этой клюшки, больше всего удаленный от центра вращения двигается с самой высокой линейной скоростью. В то же время все точки на этом отрезке движутся с одинаковой угловой скоростью. Поэтому удлиняя клюшку, биту, или ракетку, спортсмен также увеличивает линейную скорость, а соответственно скорость удара, передающуюся мячу, так что он может пролететь на большее расстояние. Укорачивая ракетку или клюшку, даже перехватив ее ниже, чем обычно, наоборот замедляют скорость удара.

При первобытнообщинном строе главными охотниками были мужчины

Спортсменам с более длинными руками и ногами удается добиться бо́льшей угловой скорости

У высоких людей с длинными конечностями есть преимущество в отношении линейной скорости. То есть, передвигая ноги с одинаковой угловой скоростью, они двигают ступни с более высокой линейной скоростью. То же происходит и с их руками. Такое преимущество может быть одной из причин того, что в первобытных обществах мужчины занимались охотой чаще, чем женщины. Вероятно, что из-за этого также в процессе эволюции выиграли более высокие люди. Длинные конечности помогали не только в беге, но и во время охоты — длинные руки бросали копья и камни с большей линейной скоростью. С другой стороны, длинные руки и ноги могут быть неудобством. Длинные конечности имеют больший вес и для их перемещения нужна дополнительная энергия. Кроме этого, когда человек быстро бежит, длинные ноги быстрее двигаются, а значит, при столкновении с препятствием удар будет сильнее, чем у людей с короткими ногами, которые двигаются с той же линейной скоростью.

В гимнастике, фигурном катании и нырянии также используют угловую скорость. Если спортсмен знает угловую скорость, то легко вычислить количество переворотов и других акробатических трюков во время прыжка. Во время кувырков спортсмены обычно прижимают ноги и руки как можно ближе к корпусу, чтобы уменьшить инерцию и увеличить ускорение, а значит и угловую скорость. С другой стороны, во время ныряния или приземления, судьи смотрят, как ровно спортсмен приземлился. На высокой скорости трудно регулировать направление полета, поэтому спортсмены специально замедляют угловую скорость, немного вытягивая от корпуса руки и ноги.

Спортсмены, которые занимаются метанием диска или молота, тоже контролируют линейную скорость с помощью угловой. Если просто бросить молот, не вращая его по кругу на длинной стальной проволоке, увеличивающей линейную скорость, то бросок будет не таким сильным, поэтому молот сначала раскручивают. Олимпийские спортсмены поворачиваются вокруг своей оси от трех до четырех раз, чтобы увеличить угловую скорость до максимально возможной.

Угловая скорость и хранение данных на оптических носителях

Диски в накопителе на жестких магнитных дисках («винчестере») вращаются со скоростями от 4&nbsp200 оборотов в минуту на портативных устройствах с низким энергопотреблением до 15&nbsp000 оборотов в минуту на высокоэффективных серверах

Во время записи данных на оптических носителях, например на компакт дисках (CD), для измерения скорости записи и считывания данных в приводе также используются угловая и линейная скорости. Существует несколько способов записи данных, во время которых используют переменную или постоянную линейную или угловую скорость. Так, например, режим постоянной линейной скорости (по-английски — Constant Linear Velocity или CVL) — один из основных методов записи дисков, при котором данные записывают с одинаковой скоростью по всей поверхности диска. Во время записи в режиме зональной постоянной линейной скорости (по-английски — Zone Constant Linear Velocity или ZCLV) постоянная скорость поддерживается во время записи на определенной части, то есть зоне диска. В этом случае диск замедляет вращение при записи на внешних зонах. Режим частично постоянной угловой скорости (Partial Constant Angular Velocity или PCAV) позволяет осуществлять запись с постепенным увеличением угловой скорости, пока она не достигнет определенного порога. После этого угловая скорость становится постоянной. Последний режим записи — режим постоянной угловой скорости (Constant Angular Velocity или CAV). В этом режиме во время записи по всей поверхности диска поддерживается одинаковая угловая скорость. При этом линейная скорость увеличивается по мере того, как записывающая головка перемещается все дальше и дальше к краю диска. Этот режим используется также при записи грампластинок и в компьютерных жестких дисках.

Угловая скорость в космосе

Геостационарная орбита

На расстоянии 35 786 километров (22 236 миль) от Земли находится орбита, на которой вращаются спутники. Это особенная орбита, потому что тела, вращающиеся на ней в одном направлении с Землей, проходят всю орбиту примерно за такое же время, которое требуется Земле, чтобы совершить полный круг вокруг своей оси. Это немного меньше 24 часов, то есть один сидерический день. Так как угловая скорость вращения тел на этой орбите равна угловой скорости вращения Земли, то наблюдателям с Земли кажется, что эти тела не движутся. Такая орбита называется геостационарной.

На эту орбиту обычно выводят спутники, которые отслеживают изменения погоды (метеорологические спутники), спутники, следящие за изменениями в океане и спутники связи, которые обеспечивают телевизионное и радиовещание, телефонную связь и спутниковый Интернет. Геостационарную орбиту часто используют для спутников потому, что антенны, один раз направленные на спутник, не нужно направлять вторично. С другой стороны, с их использованием связаны такие неудобства, как необходимость иметь прямое поле видимости между антенной и спутником. Кроме того, геостационарная орбита находится далеко от Земли и для передачи сигнала необходимо использовать более мощные передатчики, чем те, что используются для передачи с более низких орбит. Сигнал приходит с задержкой приблизительно в 0,25 секунды, что заметно для пользователей. Например, во время трансляции новостей корреспонденты в удаленных районах обычно связываются со студией по спутниковому каналу; при этом заметно, что когда телеведущий задает им вопрос, они отвечают с задержкой. Несмотря на это, спутники на геостационарной орбите широко используются. Например, до недавнего времени связь между континентами осуществлялась, главным образом, с помощью спутников. Сейчас ее в основном заменили межконтинентальные кабели, проложенные по океанскому дну; однако спутниковую связь до сих пор применяют в отдаленных районах. В последние двадцать лет спутники связи также обеспечивают доступ к интернету, особенно в отдаленных местах, где нет наземной инфраструктуры связи.

Спутниковые антенны

Срок службы спутника в основном определяется количеством топлива на борту, требуемым для периодической коррекции орбиты. Количество топлива в спутниках ограничено, поэтому когда оно заканчивается, спутники выводят из эксплуатации. Чаще всего их переводят на орбиту захоронения, то есть орбиту, намного выше геостационарной. Это — дорогостоящий процесс; однако если оставлять ненужные спутники на геостационарной орбите, это грозит вероятностью столкновений с другими спутниками. Место на геостационарной орбите ограничено, поэтому старые спутники, оставленные на орбите, будут занимать место, которое мог бы использовать новый спутник. В связи с этим во многих странах существуют нормы, требующие от владельцев спутников подписать договор о том, что в конце эксплуатации спутник будет выведен на орбиту захоронения.

Литература

Автор статьи: Kateryna Yuri

Unit Converter articles were edited and illustrated by Анатолий Золотков

Вы затрудняетесь в переводе единицы измерения с одного языка на другой? Коллеги готовы вам помочь. Опубликуйте вопрос в TCTerms и в течение нескольких минут вы получите ответ.

Расчеты для перевода единиц в конвертере «Конвертер угловой скорости и частоты вращения» выполняются с помощью функций unitconversion.org.

Скорость абразивного круга 35-50 метров в секунду это сколько оборотов/мин?

Обычно, на шлифовальных и наждачных кругах пишут линейную рабочую скорость на которую рассчитан данный круг. В характеристиках же точильных станков (наждаков) указываются рабочие обороты вала ротора электродвигателя. Казалось бы, почему производителям не прийти «к общему знаменателю» и ни озадачивать потребителя соотношением линейной скорости (метров в секунду) и угловой (оборотов в минуту) А все потому что вал ротора имеет постоянный диаметр и соответственно постоянную линейную скорость, а наждачные круги имеют как разный наружный диаметр так и разную структуру материала которая эффективно и безопасно может работать только при определенной скорости.

Вот и встает вопрос как перевести указанные допустимые пределы линейной скорости написанные на абразивных кругах, в обороты в минуту, и соотнести с возможностями своего наждака или балгарки (УШМ).

Что такое линейная скорость. В нашем случае, это сколько метров «пробежит» воображаемая точка на окружности данного абразивного круга за одну секунду. По условию вопроса, максимально возможная скорость 30 или 50 метров в секунду. На этом этапе вводится еще одна характеристика — наружный диаметр круга. Величина которая может менять линейную скорость наружной поверхности диска, при одних и тех же оборотах вала. Поэтому, при расчетах берется и длина окружности. Длина окружности находится по формуле 2ПR (2 умножаем на число П = 3,14 и умножаем на R — радиус нашего наждачного круга в метрах) Далее значение линейной скорости делим на полученное число. Но получим мы обороты в секунду, а большинство электроинструментов имеют в своих характеристиках обороты в минут, стало быть умножаем на 60.

На примере вопроса это будет выглядеть так. Возьмем диаметр наших кругов за 200 мм. (вы можете подставить и свое значение)

R — радиус такого круга будет 100 мм. а если перевести в метры то 0,1 м.

Теперь считаем длину окружности — 2 х 3,14 х 0,1 = 0,628.

Теперь рассчитываем максимально возможные обороты (или частоту вращения)

Для диска с линейной скоростью 30 м/сек.

30 / 0,628 = 47,77 оборотов в секунду. 47,77 х 60 = 2866 оборотов в минуту.

Для диска с линейной скоростью 50 м/сек.

50 / 0,628 = 79,61 оборотов в секунду. 79,61 х 60 = 4777 оборотов в минуту.

Это и есть максимальные обороты для данных дисков, с условно взятым диаметром 200 миллиметров.

1.4: Скорость и угловая скорость

Длина дуги по окружности

В разделе 1.3 мы узнали, что радианная мера угла равна длине дуги на единичной окружности, связанной с этим углом. Таким образом, дуга длины 1 на единичной окружности образует угол в 1 радиан. Бывают случаи, когда также будет полезно знать длину дуг на других кругах, которые образуют тот же угол.

Рисунок \ (\ PageIndex {1} \): Дуги, заключенные под углом в 1 радиан.

На рисунке \ (\ PageIndex {1} \) внутренний круг имеет радиус 1, внешний круг имеет радиус \ (r \), а показанный угол имеет меру \ (\ theta \) радиан. . Таким образом, длина дуги на единичной окружности, образуемой углом, равна \ (\ theta \), и мы использовали s для обозначения длины дуги на окружности радиуса \ (r \), образуемой этим углом.

Напомним, что длина окружности радиуса \ (r \) равна \ (2 \ pi r \), а длина окружности радиуса 1 равна \ (2 \ pi \).Следовательно, отношение длины дуги \ (s \) на окружности радиуса \ (r \), которая образует угол в \ (\ theta \) радиан к соответствующей дуге единичной окружности, равно \ (\ dfrac {2 \ pi r} {2 \ pi} = r \). Отсюда следует, что

\ [\ dfrac {s} {\ theta} = \ dfrac {2 \ pi r} {\ pi} \]

\ [s = r \ theta \]

Определение

На окружности радиуса \ (r \) длина s дуги, пересекаемая центральным углом с радианами, равна

.

\ [s = r \ theta \]

Примечание

Важно помнить, что для расчета длины дуги необходимо измерить центральный угол в радианах.

(Непонятно, почему буква \ (s \) обычно используется для обозначения длины дуги. Одно из объяснений состоит в том, что дуга «расширяет» угол.)

Упражнение \ (\ PageIndex {1} \)

Использование кружков в начале действия для этого раздела:

  1. Используйте формулу для длины дуги, чтобы определить длину дуги на окружности радиусом 10 футов, которая образует центральный угол в \ (\ dfrac {\ pi} {2} \) радиан. Результат равен одной четверти длины окружности?
  2. Используйте формулу для длины дуги, чтобы определить длину дуги на окружности радиусом 20 футов, которая образует центральный угол в \ (\ dfrac {\ pi} {2} \) радиан.\ circ}) = \ dfrac {11 \ pi} {90} \) и \ [s = r \ theta = (3ft) \ dfrac {11 \ pi} {90} \] \ [s = \ dfrac {11 \ pi} {30} \] Длина дуги составляет \ (\ dfrac {11 \ pi} {30} \) футов или около \ (1.1519 \) футов.

Почему радианы?

Градус знаком и удобен, так почему же мы вводим единицу радиан? Это хороший вопрос, но на него есть тонкий ответ. Как мы только что видели, длина \ (s \) дуги на окружности радиуса \ (r \), образуемой углом в \ (\ theta \) радиан, равна \ (s = r \ theta \), поэтому \ (\ theta = \ dfrac {s} {r} \).В результате радиан представляет собой отношение двух длин (отношение длины дуги к радиусу окружности), что делает радиан безразмерной величиной. Таким образом, измерение в радианах можно рассматривать как действительное число. Это удобно для работы с длиной дуги (и угловой скоростью, как мы скоро увидим), и это также будет полезно при изучении периодических явлений в главе 2. По этой причине радианная мера повсеместно используется в математике, физике и технике как в отличие от степеней, потому что, когда мы используем градусную меру, мы всегда должны учитывать градусное измерение в вычислениях.Это означает, что радиан на самом деле более естественен с математической точки зрения, чем градус.

Линейная и угловая скорость

Связь между дугой на окружности и углом, который она образует, измеряемым в радианах, позволяет нам определять величины, относящиеся к движению по окружности. Объекты, движущиеся по круговым траекториям, обладают двумя типами скорости: линейной и угловой скоростью . Представьте себе вращение на карусели. Если вы уроните камешек с края движущейся карусели, камешек не упадет прямо вниз.Вместо этого он будет продолжать двигаться вперед со скоростью, которую имела карусель в тот момент, когда камешек был выпущен. Это линейная скорость гальки. Линейная скорость измеряет, как длина дуги изменяется с течением времени.

Рассмотрим точку \ (P \), движущуюся с постоянной скоростью по окружности радиуса \ (r \). Это называется равномерным круговым движением . Предположим, что P перемещается на расстояние s единиц за время \ (t \). Линейная скорость v точки \ (P \) — это расстояние, которое она проехала, деленная на прошедшее время.То есть \ (v = \ dfrac {s} {t} \). Расстояние s — это длина дуги, и мы знаем, что \ (s = r \ theta \).

Определение: линейная скорость

Рассмотрим точку \ (P \), движущуюся с постоянной скоростью по окружности радиуса \ (r \). Линейная скорость \ (v \) точки \ (P \) равна

\ [v = \ dfrac {s} {t} = \ dfrac {r \ theta} {t} \]

где \ (\ theta \), измеренный в радианах, — это центральный угол, образованный дугой длиной \ (s \).

Другой способ измерить, насколько быстро объект движется с постоянной скоростью по круговой траектории, называется угловой скоростью. В то время как линейная скорость измеряет, как длина дуги изменяется с течением времени, угловая скорость является мерой того, насколько быстро изменяется центральный угол с течением времени.

Определение: угловая скорость

Рассмотрим точку P, движущуюся с постоянной скоростью по окружности радиуса r по дуге, соответствующей центральному углу измерения \ (\ theta \) (в радианах).Угловая скорость \ (\ omega \) точки — это радианная мера угла \ (\ theta \), деленная на время t, необходимое для того, чтобы охватить этот угол. То есть

\ [\ omega = \ dfrac {\ theta} {t}. \]

Примечание

Символ \ (\ omega \) — это строчная греческая буква «омега». Также обратите внимание, что угловая скорость не зависит от радиуса r.

Это несколько специализированное определение угловой скорости, которое немного отличается от общепринятого термина, используемого для описания скорости вращения точки по окружности.Этот срок составляет оборотов в минуту или оборотов в минуту . Иногда используется единица измерения оборотов в секунду . Лучший способ представить количество оборотов в минуту — использовать «дробь единицы» \ (\ dfrac {rev} {min} \). Поскольку 1 оборот равен \ (2 \ pi \) радиан, мы видим, что если объект min движется со скоростью x оборотов в минуту, то

\ [\ omega = x \ dfrac {rev} {min} \ cdot \ dfrac {2 \ pi rad} {rev} = x (2 \ pi) \ dfrac {rad} {min}. \]

Упражнение \ (\ PageIndex {2} \)

Предположим, круглый диск вращается со скоростью 40 оборотов в минуту.Мы хотим определить линейную скорость v (в футах в секунду) точки, находящейся на расстоянии 3 футов от центра диска.

  1. Определите угловую скорость \ (\ omega \) точки в радианах в минуту. Подсказка : Используйте формулу \ [\ omega = x \ dfrac {rev} {min} \ cdot \ dfrac {2 \ pi rad} {rev}. \]
  2. Теперь мы знаем \ (\ omega = \ dfrac {\ theta} {t} \). Поэтому используйте формулу \ (v = \ dfrac {r \ theta} {t} \), чтобы определить \ (v \) в футах в минуту.
  3. Наконец, преобразуйте линейную скорость v из футов в минуту в футы в секунду.
Ответ

1. Мы видим, что

\ [\ omega = 40 \ dfrac {rev} {min} \ times \ dfrac {2 \ pi \ space rad} {rev} \]
\ [\ omega = 80 \ pi \ dfrac {rad} {min} \ ]

2. Результат части (а) дает

\ [v = r (\ dfrac {\ theta} {r}) = r \ omega \]
\ [v = (3ft) \ times 80 \ pi \ dfrac {rad} {min} \]
\ [v = 240 \ pi \ dfrac {ft} {min} \]

3. Теперь мы переводим футы в минуту в футы в секунду.

\ [v = 240 \ pi \ dfrac {ft} {min} \ times \ dfrac {1 \ space min} {60 \ space sec} \]
\ [v = 4 \ pi \ dfrac {ft} {sec} \ около 12.566 \ dfrac {ft} {sec} \]

Обратите внимание, что в упражнении 1.18, как только мы определили угловую скорость, мы смогли определить линейную скорость. То, что мы сделали в этом конкретном случае, мы можем сделать в целом. Существует простая формула, которая напрямую связывает линейную скорость с угловой скоростью. Наша формула для линейной скорости: \ (v = \ dfrac {s} {t} \ dfrac {r \ theta} {t} \). Обратите внимание, что мы можем записать это как \ (v = r \ dfrac {\ theta} {t} \). То есть \ (v = r \ omega \)

Примечание

Рассмотрим точку \ (P \), движущуюся с постоянной (линейной) скоростью \ (v \) по окружности радиуса \ (r \).Если угловая скорость равна \ (\ omega \), то

\ [v = r \ omega \]

Итак, в упражнении 1.18, когда мы определили, что \ (\ omega = 80 \ pi \ dfrac {rad} {min} \), мы могли бы определить v следующим образом:

\ [v = r \ omega = (3 \ space ft) (80 \ pi \ dfrac {rad} {min} = 240 \ pi \ dfrac {ft} {min}). \]

Обратите внимание, что, поскольку радианы «без единиц измерения», мы можем отбросить их при работе с уравнениями, такими как предыдущее.

Пример \ (\ PageIndex {1} \): линейная и угловая скорость

LP (долгоиграющая) или виниловая пластинка со скоростью 331 об / мин — это аналоговый носитель для хранения звука, который долгое время использовался для прослушивания музыки.LP обычно имеет диаметр 12 или 10 дюймов. Чтобы работать с нашими формулами для линейной и угловой скорости, нам нужно знать угловую скорость в радианах в единицу времени. Для этого мы преобразуем \ (33 \ dfrac {1} {3} \) оборотов в минуту в радианы в минуту. Мы будем использовать тот факт, что \ (33 \ dfrac {1} {3} = \ dfrac {100} {3} \)

\ [\ omega = \ dfrac {100} {3} \ dfrac {rev} {min} \ times \ dfrac {2 \ pi \ space rad} {1 \ space rev} = \ dfrac {200 \ pi} {3 } \ dfrac {rad} {min} \]

Теперь мы можем использовать формулу v D r! для определения линейной скорости точки на краю 12-дюймовой пластинки.Радиус 6 дюймов и так

\ [v = r \ omega = (6 \ космических дюймов) (\ dfrac {200 \ pi} {3} \ dfrac {rad} {min}) = 400 \ pi \ dfrac {дюймы} {min} \]

Было бы удобнее выразить это десятичным числом в дюймах в секунду. Получаем

\ [v = 400 \ pi \ dfrac {дюймы} {мин} \ times \ dfrac {1 \ space min} {60 \ space sec} \ приблизительно 20. 944 \ dfrac {дюймы} {sec} \]

Линейная скорость составляет приблизительно 20,944 дюйма в секунду.

Упражнение \ (\ PageIndex {3} \)

Для этих задач мы предположим, что Земля представляет собой сферу с радиусом 3959 миль.Когда Земля вращается вокруг своей оси, человек, стоящий на Земле, будет путешествовать по кругу, перпендикулярному оси.

  1. Земля вращается вокруг своей оси каждые \ (24 \) часа. Определите угловую скорость Земли в радианах в час. (Оставьте свой ответ в виде числа �� \ (\ pi \).)
  2. Когда Земля вращается, человек, стоящий на экваторе, будет путешествовать по кругу с радиусом 3959 миль. Определите линейную скорость этого человека в милях в час.\ circ \) север будет двигаться по кругу радиусом 2800 миль. Определите линейную скорость этого человека в милях в час и футах в секунду.
Ответ
  1. Один оборот соответствует \ (2 \ pi \) радианам. Итак, \ [\ omega = \ dfrac {2 \ pi \ space rad} {24 \ space hr} = \ dfrac {\ pi \ space rad} {12 \ space hr}. \]
  2. Чтобы определить линейную скорость, мы используем формулу \ (v = r \ omega \) \ [v = r \ omega = (3959mi) (\ dfrac {\ pi} {12} \ dfrac {rad} {hr}) = \ dfrac {3959 \ pi} {12} \ dfrac {mi} {hr} \] Линейная скорость приблизительно равна 1036.5 миль в час.
  3. Чтобы определить линейную скорость, мы используем формулу \ (v = r \ omega \) \ [v = r \ omega = (2800mi) (\ dfrac {\ pi} {12} \ dfrac {rad} {hr}) = \ dfrac {2800 \ pi} {12} \ dfrac {mi} {hr} \] Линейная скорость составляет примерно 733,04 мили в час. Чтобы преобразовать это в футы в секунду, мы используем тот факт, что в одной миле 5280 футов, в часе 60 минут и в минуте 60 секунд. Итак,

\ [v = (\ dfrac {2800 \ pi} {12} \ dfrac {mi} {hr}) (\ dfrac {5280 \ space ft} {1 \ space mi}) (\ dfrac {1 \ space hr } {60 \ space min}) (\ dfrac {1 \ space min} {60 \ space sec}) = \ dfrac {(2800 \ pi) (5280)} {12 \ cdot 60 \ cdot 60} \ dfrac {ft } {сек} \]

Итак, линейная скорость приблизительно равна \ (1075.1 \) футов в секунду.

Сводка

В этом разделе мы изучили следующие важные концепции и идеи:

  • На окружности радиусом \ (r \) длина дуги \ (s \), пересеченная центральным углом с радианной мерой, равна \ [s = r \ theta \]
  • Равномерное круговое движение — это когда точка движется с постоянной скоростью по окружности круга. Линейная скорость — это длина дуги, пройденная точкой, деленная на прошедшее время.В то время как линейная скорость измеряет, как длина дуги изменяется во времени, угловая скорость является мерой того, насколько быстро изменяется центральный угол во времени. Угловая скорость точки — это радианная мера угла, деленная на время, необходимое для того, чтобы подметать этот угол.
  • Для точки \ (P \), движущейся с постоянной (линейной) скоростью v по окружности окружности радиуса \ (r \), имеем \ [v = r \ omega \], где \ (\ omega \) — угловая скорость точки.

единиц вращательной кинематики | Безграничная физика

Угловое положение, Тета

Угол поворота — это величина (угол) поворота фигуры относительно фиксированной точки — часто центра круга.

Цели обучения

Оценить взаимосвязь между радианами на обороте CD

Основные выводы

Ключевые моменты
  • Длина дуги Δs — это расстояние, пройденное по круговой траектории.r — радиус кривизны круговой траектории.
  • Угол поворота — это величина поворота, аналогичная линейному расстоянию. Мы определяем угол поворота [latex] \ Delta \ theta [/ latex] как отношение длины дуги к радиусу кривизны: [latex] \ Delta \ theta [/ latex] = Δs / r.
  • За один полный оборот угол поворота составляет 2π.
Ключевые термины
  • Угловое положение : угол в радианах (градусах, оборотах), на который точка или линия были повернуты в указанном направлении вокруг указанной оси.

Когда объекты вращаются вокруг некоторой оси, например, когда компакт-диск (компакт-диск) вращается вокруг своего центра, каждая точка в объекте движется по дуге окружности. Рассмотрим линию от центра компакт-диска до его края. Каждая яма, используемая для записи звука вдоль этой линии, перемещается под одним и тем же углом за одно и то же время. Угол поворота — это величина поворота, аналогичная линейному расстоянию. Мы определяем угол поворота [latex] \ Delta \ theta [/ latex] как отношение длины дуги к радиусу кривизны:

[латекс] \ Delta \ theta = \ Delta \ text {s} / \ text {r} [/ latex] (показано на).

Угол поворота : Все точки на компакт-диске перемещаются по дугам окружности. Ямки вдоль линии от центра к краю все перемещаются на один и тот же угол Δ за время Δt.

В математике угол поворота (или угловое положение) — это величина (т.е. угол), на которую фигура поворачивается относительно фиксированной точки (часто центра круга, как показано на рисунке).

Угол θ и длина дуги s : Радиус круга поворачивается на угол Δ.Длина дуги Δs указана на окружности.

Длина дуги Δs — это расстояние, пройденное по круговой траектории. r — радиус кривизны круговой траектории. Мы знаем, что за один полный оборот длина дуги равна длине окружности радиуса r. Окружность круга равна 2πr. Таким образом, за один полный оборот угол поворота составляет:

[латекс] \ Delta \ theta = (2 \ pi \ text {r}) / \ text {r} = 2 \ pi [/ latex].

Этот результат является основой для определения единиц измерения углов поворота в радианах (рад), определяемых следующим образом:

2π рад = 1 оборот.

Если [latex] \ Delta \ theta [/ latex] = 2π rad, то компакт-диск сделал один полный оборот, и каждая точка на компакт-диске вернулась в исходное положение. Поскольку в круге 360º или один оборот, отношение между радианами и градусами, таким образом, составляет 2π рад = 360º, так что:

1рад = 360º / 2π = 57,3º.

Угловая скорость, Омега

Угловая скорость ω — это скорость изменения угла, математически определяемая как ω = [latex] \ Delta \ theta [/ latex] [latex] / \ Delta \ text {t} [/ latex].

Цели обучения

Проверить, насколько быстро объект вращается на основе угловой скорости

Основные выводы

Ключевые моменты
  • Чем больше угол поворота за заданный промежуток времени, тем больше угловая скорость.
  • Угловая скорость ω аналогична линейной скорости v.
  • Мы можем записать взаимосвязь между линейной скоростью и угловой скоростью двумя разными способами: v = rω или ω = v / r.
Ключевые термины
  • угловая скорость : векторная величина, описывающая объект в круговом движении; его величина равна скорости частицы, а направление перпендикулярно плоскости ее кругового движения.

Чтобы проверить, насколько быстро объект вращается, мы определяем угловую скорость ω как скорость изменения угла. В символах это

.

[латекс] \ omega = \ Delta \ theta / \ Delta \ text {t} [/ latex],

, где угловой поворот Δ происходит за время Δt. Чем больше угол поворота за заданный промежуток времени, тем больше угловая скорость. Единицы измерения угловой скорости — радианы в секунду (рад / с).

Угловая скорость ω аналогична линейной скорости v.Чтобы найти точное соотношение между угловой и линейной скоростью, мы снова рассмотрим ямку на вращающемся CD. Эта яма перемещает дугу на длину Δs за время Δt, поэтому она имеет линейную скорость v = Δs / Δt.

Из [latex] \ Delta \ theta = (\ Delta \ text {s}) / \ text {r} [/ latex] мы видим, что [latex] \ Delta \ text {s} = \ text {r} \ cdot \ Дельта \ тета [/ латекс]. Подстановка этого в выражение для v дает [latex] \ text {v} = (\ text {r} \ cdot \ Delta \ theta) / (\ Delta \ text {t}) = \ text {r} (\ Delta \ theta / \ Delta \ text {t}) = \ text {r} \ omega [/ latex].

Мы можем записать это соотношение двумя разными способами: v = rω или ω = v / r.

Первое соотношение утверждает, что линейная скорость v пропорциональна расстоянию от центра вращения, таким образом, она является наибольшей для точки на ободе (наибольшее значение r), как и следовало ожидать. Мы также можем назвать эту линейную скорость v точки на ободе тангенциальной скоростью. Вторую взаимосвязь можно проиллюстрировать, рассмотрев шину движущегося автомобиля, как показано на рисунке ниже. Обратите внимание, что скорость точки в центре шины такая же, как скорость v автомобиля.Чем быстрее движется автомобиль, тем быстрее вращается шина — большой v означает большой ω, потому что v = rω. Точно так же шина большего радиуса, вращающаяся с той же угловой скоростью (ω), будет создавать для автомобиля большую линейную скорость (v).

Угловая скорость : Автомобиль, движущийся со скоростью v вправо, имеет шину, вращающуюся с угловой скоростью ω. Скорость протектора шины относительно оси равна v, как если бы автомобиль был поднят домкратом. Таким образом, автомобиль движется вперед с линейной скоростью v = rω, где r — радиус шины.Чем больше угловая скорость шины, тем больше скорость автомобиля.

Угловое ускорение, Alpha

Угловое ускорение — это скорость изменения угловой скорости, математически выражаемая как [latex] \ alpha = \ Delta \ omega / \ Delta \ text {t} [/ latex].

Цели обучения

Объясните взаимосвязь между угловым ускорением и угловой скоростью

Основные выводы

Ключевые моменты
  • Чем быстрее происходит изменение угловой скорости, тем больше угловое ускорение.
  • При круговом движении линейное ускорение касается окружности в интересующей точке и называется касательным ускорением.
  • При круговом движении центростремительное ускорение относится к изменению направления скорости, но не ее величины. Объект, совершающий круговое движение, испытывает центростремительное ускорение.
Ключевые термины
  • угловое ускорение : Скорость изменения угловой скорости, часто обозначаемая α.
  • тангенциальное ускорение : ускорение в направлении, касательном к окружности в интересующей точке при круговом движении.

Угловое ускорение — это скорость изменения угловой скорости. В единицах СИ он измеряется в радианах на секунду в квадрате (рад / с 2 ) и обычно обозначается греческой буквой альфа ([латекс] \ альфа [/ латекс]).

Рассмотрим следующие ситуации, в которых угловая скорость непостоянна: когда фигуристка тянет за руки, когда ребенок запускает карусель из состояния покоя или когда жесткий диск компьютера останавливается, когда он выключен.Во всех этих случаях существует угловое ускорение, при котором изменяется [латекс] \ омега [/ латекс]. Чем быстрее происходит изменение, тем больше угловое ускорение. Угловое ускорение определяется как скорость изменения угловой скорости. В форме уравнения угловое ускорение выражается следующим образом:

[латекс] \ alpha = \ Delta \ omega / \ Delta \ text {t} [/ latex]

где [latex] \ Delta \ omega [/ latex] — это изменение угловой скорости, а [latex] \ Delta \ text {t} [/ latex] — это изменение во времени.Единицы углового ускорения: (рад / с) / с или рад / с 2 . Если [latex] \ omega [/ latex] увеличивается, тогда [latex] \ alpha [/ latex] положительно. Если [latex] \ omega [/ latex] уменьшается, то [latex] \ alpha [/ latex] отрицательно.

Полезно знать, как связаны линейное и угловое ускорение. При круговом движении ускорение составляет по касательной к окружности в интересующей точке (как показано на диаграмме ниже). Это ускорение называется тангенциальным ускорением , a t .

Тангенциальное ускорение : При круговом движении ускорение может происходить из-за изменения величины скорости: a касается движения. Это ускорение называется тангенциальным ускорением.

Касательное ускорение относится к изменениям величины скорости, но не ее направления. При круговом движении центростремительное ускорение a c относится к изменениям направления скорости, но не ее величины. Объект, совершающий круговое движение, испытывает центростремительное ускорение (как показано на диаграмме ниже.) Таким образом, t и c перпендикулярны и независимы друг от друга. Касательное ускорение a t напрямую связано с угловым ускорением и связано с увеличением или уменьшением скорости (но не ее направлением).

Центростремительное ускорение : Центростремительное ускорение возникает при изменении направления скорости; он перпендикулярен круговому движению. Таким образом, центростремительное и тангенциальное ускорения перпендикулярны друг другу.

Угловая скорость

Привет, Бен.

Линейная скорость — это расстояние, пройденное по прямой за единицу времени. Угловая скорость — это угол, пройденный за единицу времени.

Безусловно, самый простой пример — это равномерное круговое движение. Например, камешек, застрявший в покрышке велосипеда, движется равномерно по кругу.

Допустим, внешний диаметр шины велосипеда составляет 70 см, и вы двигаетесь со скоростью 40 км / ч.Помните, что (игнорируя занос) шина всегда сохраняет сцепление с дорогой, поэтому расстояние, которое вы проедете, и скорость, которую вы путешествуете, совпадают со скоростью шины. Если преобразовать эту скорость в метры в секунду, мы получим:

.

Это скорость велосипеда, значит, это линейная скорость велосипеда. Поскольку байк и шина находятся в постоянном контакте, это также линейная скорость гальки в шине.

Угловая скорость связана с углами, как подсказывает название.Выражается в том, на какой угол поворачивается за определенный промежуток времени.

Углы можно измерить разными способами: вращение, градусы и радианы. Например, жесткий диск на 10000 об / мин относится к его угловой скорости: 10000 оборотов в минуту. Итак, насколько быстро вращается наш камешек?

Этот камешек, как мы знаем, движется со скоростью 11,11 м / с линейно, но движется по окружности шины. Если внешний диаметр шины d составляет 70 см, тогда длина окружности равна πd , что составляет примерно 2.199 м.

Если мы разделим линейную скорость на длину окружности шины, мы узнаем, сколько оборотов в секунду совершает эта шина (и, следовательно, галька) — под углом:

, где R = обороты.

Это угловая скорость: сколько она оборотов в единицу времени. Если вы хотите преобразовать это в градусы, просто умножьте на количество градусов на один оборот (360), если вы хотите вместо этого использовать радианы, умножьте на 2π радиан на один оборот.

А теперь попробуем другое направление. Допустим, вы знаете угловую скорость и пытаетесь вычислить соответствующую линейную скорость. Подсказка: это довольно быстро!

Возьмем для примера вращение Земли. Вы знаете, какова его угловая скорость: сколько он поворачивается в единицу времени. Очевидно, это одна ротация в день!

Давайте спросим себя, какова линейная скорость шлюза на Панамском канале. Это достаточно близко к экватору, чтобы мы могли использовать диаметр Земли на экваторе (12756 км) в качестве ориентира.Какова окружность Земли в этой точке?

Если вы вычислите длину окружности, вы можете умножить ее на угловую скорость, и вы получите линейную скорость.

Вот кое-что действительно интересное. : Поскольку угловая скорость зависит от длины окружности (и, как следствие, радиуса), у вас может быть что-то, что движется с одинаковой угловой скоростью, но с разными линейными скоростями.

Давайте подумаем о (передней) звездочке, установленной на вашем велосипеде.Если вы перейдете с маленькой звездочки на большую и будете двигать ногами с одинаковым числом оборотов в минуту (с той же угловой скоростью), то вы будете двигаться быстрее. Это потому, что линейная скорость цепи — это то, что движет скоростью велосипеда. Таким образом, когда вы меняли шестерни, вы переходили от звездочки с малым радиусом (и, следовательно, с малой окружностью) к звездочке с большим радиусом (окружностью). Когда вы умножаете эти два значения на одинаковую угловую скорость, вы получаете большую линейную скорость с большой звездочкой!

Значит, большая звездочка с двойным радиусом маленькой звездочки должна заставить вас преодолеть вдвое большее расстояние за такое же количество оборотов ног.

Суммируем:

  • Разделите линейную скорость на длину окружности, чтобы получить угловую скорость (в оборотах за единицу времени, которую вы затем можете преобразовать в любые другие единицы, которые вас интересуют).
  • Умножьте угловую скорость (в оборотах в единицу времени — сначала преобразуйте, если нужно) на длину окружности, чтобы получить линейную скорость.

Надеюсь, это проясняет это для вас!
Стивен Ла Рок>

Кинематика и динамика

Кинематика и динамика

Кинематика и динамика

Если объект вращается вокруг фиксированной оси Z с постоянным угловым ускорение α, имеем Δω = αΔt,

ω f = ω i + α (t f — t i ).

Угловое смещение θ вокруг оси z тогда равно

.

θ f = θ i + ω i (t f — t i ) + ½α (t f — t i ) 2 .

Эти уравнения имеют ту же форму, что и уравнения для линейного движения с постоянным ускорением a.
Для движения по оси абсцисс имеем

v f = v i + a (t f — t i ),
x f — x i = v xi ∆t + ½a x (t f — т и ) 2 .

Если мы заменим x на θ и a на α, то кинематическая уравнения для линейного движения вдоль оси x преобразуются в кинематические уравнения для вращательного движения вокруг оси z.

проблема:

Лайнер прибывает на аэровокзал, двигатели заглушены. выключенный. Ротор одного из двигателей имеет начальный поворот по часовой стрелке. угловая скорость 2000 рад / с. Вращение двигателей замедляется с угловое ускорение величиной 80 рад / с 2 .
(a) Определите угловую скорость через 10 с.
(b) Сколько времени нужно ротору, чтобы остановиться?

Решение:

  • Рассуждение:
    В этой задаче начальная угловая скорость ω i и угловые ускорения α даны. Если выбираем направление начального углового ускорения чтобы быть направлением z, тогда
    ω f = ω i — α (t f — t i ),
    , поскольку α находится в отрицательном направлении оси z.
  • Детали расчетов
    (a) При t = 0 ω i = 2000 / с.
    При t = 10 с имеем ω f = 2000 / с — (80 / с 2 ) (10 с) = 1200 / с.
    (б) Установка ω f = ω i — α (t f — t i ) = 0 мы можем найти время, за которое ротор придет отдыхать.
    2000 / с — (80 / с 2 ) t = 0, t = (2000/80) с = 25 с — время, необходимое ротору для остановки.
Проблема:

Вращающемуся колесу требуется 3 с для вращения 37 оборотов.Его угловатый скорость в конце 3-х секундного интервала 98 рад / с. Что постоянное угловое ускорение колеса?

Решение:

  • Рассуждение:
    Дано: Δθ = θ f — θ i = 37 оборотов, Δt = 3 с, ω f = 98 рад / с.
    С использованием θ f — θ i = ω i (t f — t i ) + ½α (t f — t i ) 2 и ω f = ω i + α (t f — t i ), у нас есть два уравнения, которые мы можем решить для двух неизвестных ω i и α.
  • Детали расчета:
    Использование θ f = θ i + ω i (t f — t i ) + ½α (t f — t i ) 2 с θ i = 0, имеем 37 * 2π = ω i * (3 с) + ½α (3 с) 2 .
    Использование ω f = ω i + α (t f — t i ) имеем 98 / s = ω i + α (3 s).
    Решаем это уравнение относительно ω i ω i = 98 с — α (3 с), и вставить это в первое уравнение.
    37 * 2π = (98 / с) (3 с) — α (3 с) 2 + ½α (3 с) 2 , 74π = 294 — α * (9 с 2 ) + α * (4,5 с 2 ),
    (4,5 с 2 ) * α = 294 — 74π, α = 13,67 / с 2 — постоянный ускорение колеса.

Когда колесо вращается вокруг оси z, каждая точка на колесе имеет одинаковый угол скорость. Однако линейная скорость v точки P зависит от расстояния до нее. от оси вращения.
Когда точка P проходит угловое смещение 2π, то его пройденное расстояние составляет 2πr.
Когда точка P проходит угловое смещение на π, то его пройденное расстояние равно πr.
Когда точка P проходит угловое смещение на θ, то пройденное расстояние составляет θr.
В терминах угловой скорости ω скорость v точки P следовательно, v = ωr, если r постоянно; v — тангенциальная скорость точки P.

Ссылка: Тангенциальная и угловая скорость (Youtube)

Касательное ускорение a точки P, движущейся по круговой траектории, приведено в с точки зрения его углового ускорения на t = rα.
Радиальное или центростремительное ускорение равно r = v 2 / r = rω 2 .
Общее ускорение равно
a = (a t 2 + a r 2 ) ½ = (r 2 α 2 + r 2 ω 4 ) ½ = r (α 2 + ω 4 ) ½ .

проблема:

Если колеса автомобиля заменены на колеса большего диаметра, показания спидометра поменять? Объяснять!

Решение:

  • Рассуждение:
    Датчик спидометра определяет угловую скорость колеса. При v номинал = r номинал ω, спидометр отображает правильная скорость, если шины имеют номинальный радиус. Если вы положите шины большего размера на вашем автомобиле, то ваша фактическая скорость v фактическая = r фактическая ω больше, чем отображаемая скорость v номинальная = r номинальная ω.
Проблема:

Автомобиль равномерно ускоряется из состояния покоя и достигает скорости 22 м / с в 9 с. Если диаметр шины составляет 58 см, найдите
(a) число оборотов, которые шина делает во время этого движения, при условии, что скольжение, и
(b) конечная скорость вращения шины в оборотах в секунду.

Решение:

  • Рассуждение:
    Дано равномерное ускорение автомобиля a = Δv / Δt. Мы можем использовать кинематические уравнения для линейного движения, чтобы найти расстояние, которое оно проходит в интервал времени Δt.
    Разделив это расстояние на окружность шины, находим число революции сделано. Конечная угловая скорость определяется по формуле ω f . = v f / r шина .
  • Детали расчета:
    (a) Ускорение автомобиля равно a = Δv / Δt = (22 м / с) / (9 с) = 2.44 м / с 2 .
    Расстояние пройденный за 9 с равен d = ½ при 2 = (½ * 2,44 * 81) м = 99 м.
    Окружность шины π * 0,58 м = 1,82 м.
    Число оборотов колеса 99 / 1,82 = 54,3.
    (b) Конечная линейная скорость шины v = 22 м / с. Используя v = ωr, ω = v / r, конечная угловая скорость составляет ω = 75,9 / с. Число оборотов в секунду составляет ω / 2π = 12 / с.

Модуль 6: Вопрос 1

Кинематические уравнения движения с постоянным линейным ускорением и такой же вид имеют движения с постоянным угловым ускорением.Сравнивать движение с постоянной скоростью с движением с постоянной угловой скоростью.

Обсудите это со своими однокурсниками на дискуссионном форуме!


Что вызывает угловое ускорение?

Предположим, вы хотите изменить вращающийся угловая скорость колеса. Для увеличения угловой скорости вы вероятно, приложит силу к ободу, касательную к ободу, и в направление мгновенной скорости сечения обода к который вы применяете силу.

Если вы хотите уменьшить угловую скорость, вы измените направление силы.

Предположим, вы хотите войти в здание с вращающейся дверцей. У двери четыре панели, и вы нажимаете на одном из них, перпендикулярном поверхности панели.
Скорость, с которой угловая скорость двери изменения, т.е. угловое ускорение α , тем больше, чем дальше от оси вращения вы применяете сила.

Угловое ускорение относительно точки является результатом крутящего момента относительно этой точки. А крутящий момент продукт рычага рычаг и сила, приложенная перпендикулярно плечу рычага. В рычаг или момент рычага — перпендикулярное расстояние от центра вращения, т.е. точка поворота к точке приложения силы.

Крутящий момент всегда определяется относительно точки поворота.

Чем больше крутящий момент, тем больше угловое ускорение. Вы можете получить больший крутящий момент, применяя большее усилие или используя более длинный рычаг. Пишем

крутящий момент = плечо рычага × сила,
τ = r × F .

Крутящий момент — вектор. это векторное произведение или перекрестное произведение из r и F .Единицы измерения крутящего момента в системе СИ — Нм.
Крутящий момент имеет величину и направление. Его направление задается Правое правило .

Пальцы правой руки должны указывать от оси вращение до точки приложения силы. Сверните их в направление F . Ваш большой палец указывает в сторону вектор крутящего момента.

Величина крутящего момента τ равно τ = rFsinθ, где θ — наименьшее угол между направлениями векторов r и F .
Мы также можем написать τ = r perp F = rF perp , где r perp — компонент плечо рычага перпендикулярно F, или где F perp — компонент F перпендикулярно плечу рычага.

Если сила F действует на объект, тогда крутящий момент, создаваемый этой силой, примерно на точка поворота: τ = r × F , где r — вектор смещения от точки поворота к точке, где сила приложена.Если на объект действуют две или более сил, то чистый крутящий момент равен векторная сумма из крутящие моменты, создаваемые отдельными силами. (Для вращения вокруг одной оси, два момента могут указывать в одном направлении или в противоположных направлениях направления и, следовательно, может складывать или вычитать.)

проблема:

На рисунке справа найдите сетку крутящий момент на колесе вокруг оси через центр, если a = 10 см и b = 25 см.

Решение:

  • Рассуждение:
    Пусть ось Z выходит из страницы. Используйте τ = rFsinθ для величины и правило правой руки для направления τ .
  • Детали расчета:
    Тогда крутящий момент, создаваемый силой 10 Н, равен τ = — (10 Н * 0,25 м) к = — (2,5 Нм) к .
    Крутящий момент, создаваемый силой 9 Н, равен τ = — (9 Н * 0.25 м) к = — (2,25 Нм) к .
    Крутящий момент, создаваемый силой 12 Н, равен τ = (12 Н * 0,1 м) к = (1,2 Нм) к .
    (Обратите внимание, что эта сила также применяется перпендикулярно плечу рычага.)
    Общий крутящий момент равен τ = — (3,55 Нм) k .

Второй закон Ньютона, когда применительно к вращательному движению утверждает, что крутящий момент равен произведение массы вращения или момент инерции I и угловое ускорение α .

крутящий момент = момент инерции × угловое ускорение
τ = I α

Момент инерции — это мера инерции вращения объекта. Это зависит от масса объекта, и как эта масса распределена относительно оси вращение. Чем дальше основная масса от оси вращения, тем больше инерция вращения (момент инерции) объекта.

Момент инерции системы относительно оси вращение можно найти, умножив массу m i каждого частица в системе на квадрат ее перпендикулярного расстояния r i от оси вращения, и суммируя все эти произведения, I = ∑m i r i 2 .

проблема:

Четыре частицы на рисунке справа связаны жесткими стержнями. Начало координат находится в центре прямоугольника.Рассчитайте момент инерции системы относительно оси z.

Решение:

  • Рассуждение:
    Момент инерции I = ∑m i r i 2 . Здесь r i — перпендикулярное расстояние частицы i от ось z.
  • Детали расчета:
    Каждая частица — это расстояние r = (9 + 4) ½ м = (13) ½ м от ось вращения.
    I = (3 кг + 2 кг + 4 кг + 2 кг) * 13 м 2 = 143 кг · м 2 .

Момент инерции объекта — это мера его сопротивления угловому ускорению. Из-за его инерции вращения вам необходим крутящий момент, чтобы изменить угловую скорость объект. Если на объект не действует крутящий момент, его угловая скорость не изменится. Если он изначально не вращается, он не начнет вращаться. Если он крутится с заданным угловая скорость, эта угловая скорость не изменится.И его угловая скорость, и ориентация его оси вращения останется прежней.

постоянная угловая скорость <--> без угловой ускорение <--> нет чистого крутящего момента

Когда на два объекта действует один и тот же крутящий момент, объект с большим моментом инерции имеет меньший угловой ускорение. Единицы момента инерции: единицы массы, умноженные на квадрат расстояния, например кгм 2 .

Представьте себе два колеса одинаковой массы.Одно из них — сплошное колесо с его масса равномерно распределена по всей конструкции, в то время как другая имеет большая часть массы сосредоточена у обода.

Колесо с массой около обода имеет больший момент инерция.

Момент инерции определяется относительно оси вращения.

Например, момент инерции кругового диска, вращающегося вокруг оси через свой центр, перпендикулярный плоскости диска, отличается от момент инерции диска, вращающегося вокруг оси через его центр в плоскости диска.

Ссылка: Список моментов инерции


Качели

Двое детей играют на качелях, раскачиваясь взад и вперед. Центр качели зафиксированы. Поступательного движения нет. Мы наблюдаем ротационные движение вокруг центра. Пока качели движутся, практически нет крутящий момент на качелях, и он вращается с равномерной угловой скоростью. Вес каждого ребенка умножает рычаг от центра до того места, где сидит ребенок создает крутящий момент, но два крутящих момента имеют одинаковую величину и точку в в противоположных направлениях и поэтому отменить.Если один ребенок тяжелее, он садится ближе к центру, чем более легкий ребенок. Это уменьшает плечо рычага. В таким образом, разные веса могут создавать крутящие моменты одинаковой величины.

Когда ступня одного ребенка ударяется об пол, пол отталкивается и производит крутящий момент, направленный противоположно угловой скорости. Этот крутящий момент уменьшит угловая скорость до нуля за короткий промежуток времени. Качели остановлены. Затем ребенок отталкивается, а земля отталкивается.Направление крутящий момент остается прежним. Крутящий момент вызывает угловое ускорение, в результате чего с угловой скоростью, противоположной первоначальному направлению. Качели достигает своей конечной угловой скорости, когда ребенок перестает толкать. Теперь он сохраняет при вращении с постоянной угловой скоростью, пока стопы другого ребенка не коснутся пол.

Ссылка: Качели

Движение по кругу — Математика A-Level, редакция

Эта страница описывает движение по кругу.

Угловая скорость

Представьте, что объект движется по круговой траектории.

Угловая скорость — это скорость изменения угла (который я обозначил буквой «а»). Таким образом, он измеряет, насколько быстро объект движется по кругу.

Угловая скорость обычно измеряется в радиан, в секунду (рад с -1 ), то есть на сколько радиан проходит частица за секунду. Кроме того, его можно измерить в оборотах в секунду, т.е. сколько полных кругов объект проходит за секунду.

Существует формула, соединяющая «нормальную» скорость (обычно называемую «линейной скоростью») и угловую скорость:

где v — линейная скорость, r — радиус окружности, а w — угловая скорость.

Пример

Частица движется по окружности радиусом 10 см. Угловая скорость 2 рад с -1 . Найдите (линейную) скорость.

Нам нужен радиус в метрах, то есть 0,1 м. Используя приведенную выше формулу, получаем:

v = 0.1 × 2 = 0,2

Значит скорость 0,2 мс -1 .

Обратите внимание, что если вам дана угловая скорость в оборотах в секунду, вам нужно сначала преобразовать в радианы в секунду. Для этого помните, что 1 оборот в секунду равен 2p радианам в секунду, потому что в круге 2p радиан.

Радиальное ускорение

Если тело движется по кругу, даже если оно движется с постоянной скоростью, оно ускоряется.Это потому, что он меняет направление (не движется по прямой).

Направление этого ускорения — к центру круга, а его величина определяется выражением:

где v — скорость, а r — радиус окружности.

Используя нашу формулу выше, это также можно записать как:

Какие из них вы будете использовать, будет зависеть от того, имеете ли вы дело со скоростью или угловой скоростью.

Ускорение происходит из-за действующей силы:

Представьте себе, что вы едете в машине, быстро объезжающей поворот налево.Вы почувствуете, как сила тянет вас в сторону (левую сторону). Это сила, вызывающая ускорение. Сила действует по направлению к центру круга.

Конический маятник

Конический маятник выглядит примерно так:

P — частица. AP — это строка. P движется по синему кругу с угловой скоростью w.

Пример

Предположим, у нас есть конический маятник, как указано выше, где частица имеет массу 2 кг, а радиус круга, по которому движется частица, равен 0.5 м, а угол А составляет 45 градусов. Найти угловую скорость P.

Масса 2 г (W = мг), где g — ускорение свободного падения.

Вертикальное разрешение: Tcos45 = 2g
Следовательно (√2T) / 2 = 2g, поэтому T = 2√2 g (1)

Теперь используйте 2-й закон Ньютона, чтобы найти уравнение движения в радиальном направлении:
(«F = m r w 2 «)
Tsin45 = 2 × 5 × w 2

Используйте (1), чтобы исключить T:
2√2 g × (√2) / 2 = 10w 2
g / 5 = w 2
Таким образом, w = √ (g / 5)

Принимая g = 9.8, мы находим, что угловая скорость составляет 1,4 рад с -1

Движение на насыпной поверхности

Теперь рассмотрим движение частицы по «наклонной поверхности». Под этим я, например, имею в виду кольцевую гоночную трассу, которая наклонена вверх от центра, чтобы помочь автомобилям / мотоциклам оставаться на трассе на высоких скоростях.

Так вот, если машина едет очень быстро, она будет скользить по склону, двигаясь по кругу. Если он будет двигаться медленно, он поскользнется.

Если автомобиль не имеет тенденции к скольжению, силы и ускорение, действующие на кузов, будут такими, как на этой диаграмме (сила трения отсутствует):

Однако, если бы машина двигалась быстрее, она бы соскользнула по склону при движении по трассе. Таким образом, сила трения будет действовать, пытаясь предотвратить это:

Решите задачи угловой скорости — Precalculus

Если вы считаете, что контент, доступный через Веб-сайт (как определено в наших Условиях обслуживания), нарушает или другие ваши авторские права, сообщите нам, отправив письменное уведомление («Уведомление о нарушении»), содержащее в информацию, описанную ниже, назначенному ниже агенту.Если репетиторы университета предпримут действия в ответ на ан Уведомление о нарушении, оно предпримет добросовестную попытку связаться со стороной, которая предоставила такой контент средствами самого последнего адреса электронной почты, если таковой имеется, предоставленного такой стороной Varsity Tutors.

Ваше Уведомление о нарушении прав может быть отправлено стороне, предоставившей доступ к контенту, или третьим лицам, таким как в качестве ChillingEffects.org.

Обратите внимание, что вы будете нести ответственность за ущерб (включая расходы и гонорары адвокатам), если вы существенно искажать информацию о том, что продукт или действие нарушает ваши авторские права.Таким образом, если вы не уверены, что контент находится на Веб-сайте или по ссылке с него нарушает ваши авторские права, вам следует сначала обратиться к юристу.

Чтобы отправить уведомление, выполните следующие действия:

Вы должны включить следующее:

Физическая или электронная подпись правообладателя или лица, уполномоченного действовать от их имени; Идентификация авторских прав, которые, как утверждается, были нарушены; Описание характера и точного местонахождения контента, который, по вашему мнению, нарушает ваши авторские права, в \ достаточно подробностей, чтобы позволить репетиторам университетских школ найти и точно идентифицировать этот контент; например нам требуется а ссылка на конкретный вопрос (а не только на название вопроса), который содержит содержание и описание к какой конкретной части вопроса — изображению, ссылке, тексту и т. д. — относится ваша жалоба; Ваше имя, адрес, номер телефона и адрес электронной почты; а также Ваше заявление: (а) вы добросовестно считаете, что использование контента, который, по вашему мнению, нарушает ваши авторские права не разрешены законом, владельцем авторских прав или его агентом; (б) что все информация, содержащаяся в вашем Уведомлении о нарушении, является точной, и (c) под страхом наказания за лжесвидетельство, что вы либо владелец авторских прав, либо лицо, уполномоченное действовать от их имени.

Отправьте жалобу нашему уполномоченному агенту по адресу:

Чарльз Кон Varsity Tutors LLC
101 S. Hanley Rd, Suite 300
St. Louis, MO 63105

Или заполните форму ниже:

10.2 Кинематика вращательного движения — Физика колледжа, главы 1-17

Сводка

  • Соблюдайте кинематику вращательного движения.
  • Выведите кинематические уравнения вращения.
  • Оценить стратегии решения проблем для вращательной кинематики.

Просто используя нашу интуицию, мы можем начать видеть, как вращательные величины, такие как [латекс] \ boldsymbol {\ theta}, \: \ boldsymbol {\ omega}, [/ latex] и [латекс] \ boldsymbol {\ alpha} [ / latex] связаны друг с другом. Например, если колесо мотоцикла имеет большое угловое ускорение в течение довольно длительного времени, оно быстро вращается и совершает много оборотов. Говоря более техническим языком, если угловое ускорение колеса [латекс] \ boldsymbol {\ alpha} [/ latex] велико в течение длительного периода времени [латекс] \ boldsymbol {t}, [/ latex], то конечная угловая скорость [ latex] \ boldsymbol {\ omega} [/ latex] и угол поворота [latex] \ boldsymbol {\ theta} [/ latex] большие.Вращательное движение колеса в точности аналогично тому, что большое поступательное ускорение мотоцикла дает большую конечную скорость, и пройденное расстояние также будет большим.

Кинематика — это описание движения. Кинематика вращательного движения описывает отношения между углом поворота, угловой скоростью, угловым ускорением и временем. Давайте начнем с поиска уравнения, связывающего [латекс] \ boldsymbol {\ omega}, \: \ boldsymbol {\ alpha}, [/ latex] и [латекс] \ boldsymbol {t}.[/ latex] Чтобы определить это уравнение, вспомним известное кинематическое уравнение для поступательного или прямолинейного движения:

[латекс] \ boldsymbol {v = v_0 + at \ textbf {(константа a)}} [/ латекс]

Обратите внимание, что во вращательном движении [латекс] \ boldsymbol {a = a _ {\ textbf {t}}}, [/ latex] и мы будем использовать символ [латекс] \ boldsymbol {a} [/ latex] для тангенциального или линейного ускорение с этого момента. Как и в линейной кинематике, мы предполагаем, что [латекс] \ boldsymbol {a} [/ latex] является постоянным, что означает, что угловое ускорение [латекс] \ boldsymbol {\ alpha} [/ latex] также является постоянным, потому что [латекс] \ жирный символ {a = r \ alpha}.[/ latex] Теперь давайте заменим [latex] \ boldsymbol {v = r \ omega} [/ latex] и [latex] \ boldsymbol {a = r \ alpha} [/ latex] в приведенное выше линейное уравнение:

[латекс] \ boldsymbol {r \ omega = r \ omega_0 + r \ alpha {t}}. [/ Latex]

Радиус [латекс] \ boldsymbol {r} [/ latex] сокращается в уравнении, давая

[латекс] \ boldsymbol {\ omega = \ omega_0 + at \ textbf {(константа a),}} [/ latex]

где [latex] \ boldsymbol {\ omega_0} [/ latex] — начальная угловая скорость. Это последнее уравнение представляет собой кинематическое соотношение между [latex] \ boldsymbol {\ omega}, \: \ boldsymbol {\ alpha}, [/ latex] и [latex] \ boldsymbol {t} [/ latex], т. Е. он описывает их отношения без ссылки на силы или массы, которые могут повлиять на вращение.Он также точно аналогичен по форме своему трансляционному аналогу.

ПОДКЛЮЧЕНИЕ

Кинематика вращательного движения полностью аналогична поступательной кинематике, впервые представленной в главе 2 «Одномерная кинематика». Кинематика занимается описанием движения без учета силы или массы. Мы обнаружим, что поступательные кинематические величины, такие как смещение, скорость и ускорение, имеют прямые аналоги во вращательном движении.

Исходя из четырех кинематических уравнений, которые мы разработали в главе 2 «Одномерная кинематика», мы можем вывести следующие четыре кинематических уравнения вращения (представленные вместе с их аналогами для поступательного движения):

Поворотный Трансляционный
[латекс] \ boldsymbol {\ theta = \ bar {\ omega} t} [/ latex] [латекс] \ boldsymbol {x = \ bar {v} t} [/ latex]
[латекс] \ boldsymbol {\ omega = \ omega_0 + \ alpha {t}} [/ латекс] [латекс] \ boldsymbol {v = v_0 + at} [/ латекс] (константа [латекс] \ boldsymbol {\ alpha, \: a} [/ latex])
[латекс] \ boldsymbol {\ omega = \ omega_0 {t} + \ frac {1} {2} \ alpha {t} ^ 2} [/ латекс] [латекс] \ boldsymbol {x = v_0t + \ frac {1} {2} at ^ 2} [/ latex] (константа [латекс] \ boldsymbol {\ alpha, \: a} [/ latex])
[латекс] \ boldsymbol {\ omega ^ 2 = \ omega_0 ^ 2 + 2 \ alpha \ theta} [/ латекс] [латекс] \ boldsymbol {v ^ 2 = v_0 ^ 2 + 2ax} [/ латекс] (константа [латекс] \ boldsymbol {\ alpha, \: a} [/ latex])
Таблица 2. Вращательные кинематические уравнения.

В этих уравнениях индекс 0 обозначает начальные значения ([latex] \ boldsymbol {\ theta_0}, \: \ boldsymbol {x_0}, [/ latex] и [latex] \ boldsymbol {t_0} [/ latex] — начальные значения), а средняя угловая скорость [латекс] \ boldsymbol {\ bar {\ omega}} [/ latex] и средняя скорость [латекс] \ boldsymbol {\ bar {v}} [/ latex] определяются следующим образом :

[латекс] \ boldsymbol {\ bar {\ omega} \: =} [/ latex] [латекс] \ boldsymbol {\ frac {\ omega_0 + \ omega} {2}} [/ latex] [латекс] \ textbf {и } \ boldsymbol {\ bar {v} \: =} [/ latex] [латекс] \ boldsymbol {\ frac {v_0 + v} {2}}.[/ латекс]

Уравнения, приведенные выше в таблице 2, могут использоваться для решения любой задачи кинематики вращения или поступательного движения, в которой [latex] \ boldsymbol {a} [/ latex] и [latex] \ boldsymbol {\ alpha} [/ latex] являются постоянными.

СТРАТЕГИЯ РЕШЕНИЯ ПРОБЛЕМ ДЛЯ ВРАЩАТЕЛЬНОЙ КИНЕМАТИКИ


  1. Изучите ситуацию, чтобы определить, задействована ли кинематика вращения (вращательное движение) . Должно быть задействовано вращение, но без учета сил или масс, влияющих на движение.
  2. Определите, что именно необходимо определить в проблеме (определите неизвестные) . Набросок ситуации полезен.
  3. Составьте список того, что дано или может быть выведено из проблемы, как указано (определить известные) .
  4. Решите соответствующее уравнение или уравнения для определяемой величины (неизвестное значение) . Может быть полезно думать в терминах трансляционного аналога, потому что теперь вы знакомы с таким движением.
  5. Подставьте известные значения вместе с их единицами измерения в соответствующее уравнение и получите численные решения вместе с единицами измерения . Обязательно используйте радианы для углов.
  6. Проверьте свой ответ, чтобы убедиться, что он разумен: Имеет ли смысл ваш ответ ?

Пример 1: Расчет ускорения рыболовной катушки

Глубоководный рыбак ловит большую рыбу, которая отплывает от лодки, выдергивая леску из своей рыболовной катушки.2} [/ latex] в течение 2,00 с, как показано на рисунке 1.

(а) Какова конечная угловая скорость мотовила?

(b) С какой скоростью леска покидает катушку по истечении 2,00 с?

(c) Сколько оборотов делает катушка?

(d) Сколько метров лески сошло с катушки за это время?

Стратегия

В каждой части этого примера стратегия такая же, как и для решения задач линейной кинематики. В частности, идентифицируются известные значения и затем ищется взаимосвязь, которая может использоваться для определения неизвестного.

Решение для (а)

Здесь даны [латекс] \ boldsymbol {\ alpha} [/ latex] и [latex] \ boldsymbol {t} [/ latex] и необходимо определить [latex] \ boldsymbol {\ omega} [/ latex]. Наиболее простое в использовании уравнение — [латекс] \ boldsymbol {\ omega = \ omega_0 + \ alpha {t}} [/ latex], потому что неизвестное уже на одной стороне, а все остальные термины известны. Это уравнение утверждает, что

[латекс] \ boldsymbol {\ omega = \ omega_0 + \ alpha {t}. 2) (2.00 \ textbf {s}) = 220 \ textbf {рад / с.}} [/ Latex]

Решение для (b)

Теперь, когда [латекс] \ boldsymbol {\ omega} [/ latex] известен, скорость [латекс] \ boldsymbol {v} [/ latex] проще всего определить с помощью соотношения

[латекс] \ boldsymbol {v = r \ omega,} [/ латекс]

, где радиус [латекс] \ boldsymbol {r} [/ латекс] катушки задан равным 4,50 см; таким образом,

[латекс] \ boldsymbol {v = (0,0450 \ textbf {m}) (220 \ textbf {rad / s}) = 9.90 \ textbf {m / s.}} [/ Latex]

Еще раз обратите внимание, что радианы всегда должны использоваться в любых вычислениях, касающихся линейных и угловых величин.Кроме того, поскольку радианы безразмерны, мы имеем [latex] \ boldsymbol {\ textbf {m} \ times \ textbf {rad} = \ textbf {m}}. [/ Latex]

Решение для (c)

Здесь нас просят найти количество оборотов. Поскольку [latex] \ boldsymbol {1 \ textbf {rev} = 2 \ pi \ textbf {rad}}, [/ latex], мы можем узнать количество оборотов, найдя [latex] \ boldsymbol {\ theta} [/ latex] в радианах. 2}.2 = 220 \ textbf {рад.}} \ End {array} [/ latex]

Преобразование радианов в обороты дает

[латекс] \ boldsymbol {\ theta = (220 \ textbf {rad})} [/ latex] [латекс] \ boldsymbol {\ frac {1 \ textbf {rev}} {2 \ pi \ textbf {rad}}} [/ latex] [latex] \ boldsymbol {= 35.0 \ textbf {rev.}} [/ latex]

Решение для (d)

Длина лески [latex] \ boldsymbol {x}, [/ latex] в метрах может быть получена через ее связь с [latex] \ boldsymbol {\ theta}: [/ latex]

[латекс] \ boldsymbol {x = r \ theta = (0.0450 \ textbf {m}) (220 \ textbf {rad}) = 9.90 \ textbf {m}}. [/ Latex]

Обсуждение

Этот пример показывает, что отношения между вращательными величинами очень похожи на отношения между линейными величинами. Мы также видим в этом примере, как связаны линейные и вращательные величины. Ответы на вопросы реалистичны. После раскручивания в течение двух секунд катушка вращается со скоростью 220 рад / с, что составляет 2100 об / мин. 2}} [/ latex] [латекс] \ boldsymbol {= 0.733 \ textbf {s.}} [/ Latex]

Обсуждение

Обратите внимание, что следует проявлять осторожность со знаками, указывающими направление различных величин. Также обратите внимание, что время остановки барабана довольно мало, потому что ускорение довольно велико. Леска иногда ломается из-за участвующих в ней ускорений, и рыбаки часто позволяют рыбе плавать некоторое время, прежде чем тормозить катушку. Уставшая рыба будет медленнее, требуя меньшего ускорения.

Пример 3: Расчет медленного ускорения поездов и их колес

Большие грузовые поезда очень медленно ускоряются.2}. [/ Latex] После того, как колеса сделали 200 оборотов (предположим, что проскальзывания нет): (a) Как далеко поезд продвинулся по рельсам? б) Какова конечная угловая скорость колес и линейная скорость поезда?

Стратегия

В части (a) нас просят найти [latex] \ boldsymbol {x}, [/ latex], а в (b) нас просят найти [latex] \ boldsymbol {\ omega} [/ latex] и [ латекс] \ boldsymbol {v}. [/ latex] Нам даны количество оборотов [латекс] \ boldsymbol {\ theta}, [/ latex] радиус колес [латекс] \ boldsymbol {r}, [/ латекс ] и угловое ускорение [латекс] \ boldsymbol {\ alpha}.[/ латекс]

Решение для (а)

Расстояние [латекс] \ boldsymbol {x} [/ latex] очень легко найти из соотношения между расстоянием и углом поворота:

[латекс] \ boldsymbol {\ theta \: =} [/ latex] [латекс] \ boldsymbol {\ frac {x} {r}}. [/ Latex]

Решение этого уравнения для [latex] \ boldsymbol {x} [/ latex] дает

[латекс] \ boldsymbol {x = r \ theta}. [/ Latex]

Перед использованием этого уравнения мы должны преобразовать количество оборотов в радианы, потому что мы имеем дело с соотношением между линейными и вращательными величинами:

[латекс] \ boldsymbol {\ theta = (200 \ textbf {rev})} [/ latex] [латекс] \ boldsymbol {\ frac {2 \ pi \ textbf {rad}} {1 \ textbf {rev}}} [/ латекс] [латекс] \ boldsymbol {= 1257 \ textbf {рад. 2 + 2 \ alpha \ theta} [/ латекс]

Извлечение квадратного корня из этого уравнения и ввод известных значений дает

[латекс] \ begin {array} {lcl} \ boldsymbol {\ omega} & \ boldsymbol {=} & \ boldsymbol {[0 + 2 (0.{1/2}} \\ {} & \ boldsymbol {=} & \ boldsymbol {25.1 \ textbf {rad / s.}} \ End {array} [/ latex]

Мы можем найти линейную скорость поезда, [latex] \ boldsymbol {v}, [/ latex] через его связь с [latex] \ boldsymbol {\ omega}: [/ latex]

[латекс] \ boldsymbol {v = r \ omega = (0.350 \ textbf {m}) (25.1 \ textbf {rad / s}) = 8.77 \ textbf {m / s.}} [/ Latex]

Обсуждение

Пройденное расстояние довольно велико, а конечная скорость довольно мала (чуть менее 32 км / ч).

Существует поступательное движение даже для чего-то, вращающегося на месте, как показано в следующем примере. На рис. 2 изображена муха на краю вращающейся пластины микроволновой печи. В приведенном ниже примере вычисляется общее пройденное расстояние.

Рис. 2. На изображении показана микроволновая пластина. Муха совершает обороты, пока еда разогревается (вместе с мухой).

Пример 4: Расчет расстояния, пройденного мухой на краю плиты микроволновой печи

Человек решает использовать микроволновую печь, чтобы разогреть обед.При этом муха случайно влетает в микроволновку, приземляется на внешний край вращающейся пластины и остается там. Если тарелка имеет радиус 0,15 м и вращается со скоростью 6,0 об / мин, рассчитайте общее расстояние, пройденное мухой за 2,0-минутный период приготовления. (Игнорируйте время запуска и замедления.)

Стратегия

Сначала найдите общее количество оборотов [latex] \ boldsymbol {\ theta}, [/ latex], а затем линейное расстояние [latex] \ boldsymbol {x} [/ latex], пройденное.[latex] \ boldsymbol {\ theta = \ bar {\ omega} t} [/ latex] можно использовать для поиска [latex] \ boldsymbol {\ theta} [/ latex], потому что [latex] \ boldsymbol {\ bar {\ omega}} [/ latex] задано 6,0 об / мин.

Решение

Ввод известных значений в [latex] \ boldsymbol {\ theta = \ bar {\ omega} t} [/ latex] дает

[латекс] \ boldsymbol {\ theta = \ bar {\ omega} t = (6.0 \ textbf {rpm}) (2.0 \ textbf {min}) = 12 \ textbf {rev}.} [/ Latex]

Как всегда, необходимо преобразовать обороты в радианы перед вычислением линейной величины, такой как [latex] \ boldsymbol {x} [/ latex], из угловой величины, такой как [latex] \ boldsymbol {\ theta}: [/ latex]

[латекс] \ boldsymbol {\ theta = (12 \ textbf {rev})} [/ latex] [латекс] \ boldsymbol {\ frac {2 \ pi \ textbf {rad}} {1 \ textbf {rev}}} [/ латекс] [латекс] \ boldsymbol {= 75.4 \ textbf {rad}.} [/ Latex]

Теперь, используя соотношение между [latex] \ boldsymbol {x} [/ latex] и [latex] \ boldsymbol {\ theta}, [/ latex], мы можем определить пройденное расстояние:

[латекс] \ boldsymbol {x = r \ omega = (0.15 \ textbf {m}) (75.4 \ textbf {rad}) = 11 \ textbf {m}}. [/ Latex]

Обсуждение

Неплохая поездка (если выживет)! Обратите внимание, что это расстояние — это общее расстояние, пройденное мухой. Смещение фактически равно нулю для полных оборотов, потому что они возвращают муху в исходное положение.Различие между общим пройденным расстоянием и перемещением было впервые отмечено в главе 2 «Одномерная кинематика».

Проверьте свое понимание

1: Кинематика вращения имеет множество полезных взаимосвязей, часто выражаемых в форме уравнений. Являются ли эти отношения законами физики или они просто описательны? (Подсказка: тот же вопрос относится к линейной кинематике.)

Задачи и упражнения

1: С помощью струны гироскоп ускоряется из состояния покоя до 32 рад / с за 0.40 с.

(а) Каково его угловое ускорение в рад / с 2 ?

(б) Сколько оборотов он совершает в процессе?

2: Предположим, на компакт-диске оказался кусок пыли. Если скорость вращения компакт-диска составляет 500 об / мин, а пылинка находится на расстоянии 4,3 см от центра, какое общее расстояние проходит пыль за 3 минуты? (Игнорируйте ускорения из-за вращения компакт-диска.)

3: Гироскоп замедляется с начальной скорости 32.2}. [/ Латекс]

(a) Каково угловое ускорение его шин радиусом 0,280 м, если предположить, что они не скользят по тротуару?

(b) Сколько оборотов делают шины перед остановкой, учитывая их начальную угловую скорость [латекс] \ boldsymbol {95.0 \ textbf {rad / s}} [/ latex]?

(c) Сколько времени нужно автомобилю, чтобы полностью остановиться?

(d) Какое расстояние машина проезжает за это время?

(e) Какова была начальная скорость автомобиля?

(f) Кажутся ли полученные значения разумными, учитывая, что эта остановка происходит очень быстро?

Рисунок 3.2}, [/ latex] какое угловое ускорение у йо-йо?

(б) Какова угловая скорость через 0,750 с, если она начинается из состояния покоя?

(c) Внешний радиус йо-йо составляет 3,50 см. Каково тангенциальное ускорение точки на краю?

Глоссарий

кинематика вращательного движения
описывает отношения между углом поворота, угловой скоростью, угловым ускорением и временем.

Добавить комментарий

Ваш адрес email не будет опубликован. Обязательные поля помечены *